Chemistry SQP XII

You might also like

Download as pdf or txt
Download as pdf or txt
You are on page 1of 106

SAMPLE QUESTION PAPER

CHEMISTRY (043)

CLASS – XII – (2014-2015)

QUESTION PAPER DESIGN


CLASS XII CBSE

Types of Questions

S.No Type of Question Marks for No. of Total Marks


each Questions
Question
1. Long Answers (LA) 5 03 15
2. Value based question 4 01 04
3. Short Answers-II(SA II) 3 12 36
4. Short Answers-I(SA I) 2 05 10
5. Very Short Answer (VSA) 1 05 05
Total 26 70

Type of questions VSA(1) SA1(2) SA2(3) VB(4) LA(5) Total(%)


Knowledge 2 1 1 - - 07 (10%)
Understanding - 2 4 - 1 21 (30%)
Application - 2 4 - 1 21 (30%)
HOTS (Higher order thinking skills) 2 - 1 - 1 10(14%)
Evaluation and Multi-disciplinary 1 - 2 1 - 11(16%)

Page 1 of 10
SAMPLE QUESTION PAPER

CHEMISTRY

CLASS – XII (2014 – 2015)

Time Allowed: 3 hr Maximum marks: 70

General Instructions:
(a) All questions are compulsory.
(b) Q.no. 1 to 5 are very short answer questions and carry 1 mark each.
(c) Q.no. 6 to 10 are short answer questions and carry 2 marks each.
(d) Q.no. 11 to 22 are also short answer questions and carry 3 marks each
(e) Q.no. 23 is a value based question and carry 4 marks.
(f) Q.no. 24 to 26 are long answer questions and carry 5 marks each
(g) Use log tables if necessary, use of calculators is not allowed.

1. The following figure shows the variation of adsorption of N2 on charcoal with pressure at
different constant temperatures:

Page 2 of 10
Arrange the temperatures T1, T2 and T3 in the increasing order.

2. Give the formula of a noble gas species which is isostructural with IBr2-.
3. What is the effect of synergic bonding interactions in a metal carbonyl complex?
4. PCl5 acts as an oxidizing agent. Justify.
5. Write the name of the product formed when benzenediazonium chloride solution is treated with
potassium iodide.
6. Name the crystal defect which reduces the density of an ionic solid? What type of ionic
substances show this defect?
7. The molar conductivity ( λ ) of KCl solutions at different concentrations at 298 K is plotted as
m
shown in the figure given below:

Determine the value of λ0m and A for KCl.


8. Aluminum crystallizes in anfcc structure. Atomic radius of the metal is 125 pm. What is the
length of the side of the unit cell of the metal?
9. Draw the structure of the following compounds:
(i) H2S2O7
(ii) XeOF4

OR

Write balanced chemical equations for the following:


(i) Reaction of chlorine with hot and concentrated NaOH.

Page 3 of 10
(ii) Sulphur dioxide is passed through an aqueous solution of Fe (III) salt.
10. 0.5 g of KCl was dissolved in 100 g of water and the solution originally at 200C, froze at
-0.240C. Calculate the percentage dissociation of the salt.
(Given :Kf for water = 1.86 K kg /mol, Atomic mass: K = 39 u, Cl= 35.5 u)
11. State briefly the principles involved in the following operations in metallurgy. Give an
example.
(i) Hydraulic washing.
(ii) Zone refining.
12.
i) What type of deviation from Raoult’s law is observed, when two volatile liquids A and
B on mixing produce a warm solution? Explain with the help of a well labeled vapour
pressure graph.
ii) Consider separate solutions of 0.5 M CH3OH, 0.250 M KCl (aq) and 0.125 M Na3PO4
(aq). Arrange the above solutions in the increasing order of their Van’t Hoff factor.

13. Write the Nernst equationand calculate the emffor the following cell at 298 K:
Mg(s) / Mg2+ (0.001 M) // Cu2+ (0.0001 M) / Cu(s)
How does Ecellvary with the concentration of both Mg2+ and Cu2+ ions?
(GivenEocell= 2.71 V)
14. Explain the following observations giving appropriate reasons:
(i) Ozone is thermodynamically unstable with respect to oxygen. .
(ii) The HEH bond angle of the hydrides of group 15 elements decrease as we move down
the group.
(iii) Bleaching effect of chlorine is permanent.
15.
(i) Predict the number of unpaired electrons in the tetrahedral [MnBr4]2- ion.
(ii) Draw structures of geometrical isomers of [Co(NH3)4Cl2]+.
(iii) Write the formula for the following coordinate compound:
Amminebromidochloridonitrito-N-platinate(II)
16. Explain what is observed when
(i) Silver nitrate solution is added to potassium iodide solution.
(ii) The size of the finest gold sol particles increases in the gold sol.
(iii) Two oppositely charged sols are mixed in almost equal proportions.

Page 4 of 10
17.
(i) In the following pairs of halogen compounds, which would undergo SN1 reaction
faster? Explain.
Cl
Cl

and

(ii) Amongst the isomeric dihalobenzenes which isomer has the highest melting point and
why?
(iii) Arrange the following haloalkanes in the increasing order of density. Justify your
answer.
CCl4, CH2Cl2 and CHCl3.

18. An organic compound ( A ) has characteristic odour. On treatment with NaOH, it forms
compounds ( B ) and ( C ). Compound ( B ) has molecular formula C7H 8Owhich on oxidation
gives back ( A ). The compound ( C ) is a sodium salt of an acid. When ( C ) is treated with
soda-lime, it yields an aromatic compound ( D ). Deduce the structures of ( A ), ( B ), ( C ) and
( D ). Write the sequence of reactions involved.
19. (a) Give one chemical test to distinguish between the following pairs of compounds:
(i) Methylamine and dimethylamine.
(ii) Aniline and benzylamine
(b) Write the structures of different isomers corresponding to the molecular formula C3H9N,
which will liberate nitrogen gas on treatment with nitrous acid.
20. (a) Exemplify the following reactions:
(i) Rosenmund reduction reaction.
(ii) Kolbe electrolysis reaction.
(b) Arrange the following compounds in increasing order of their reactivity towards HCN:
Acetaldehyde, Acetone, Di-tert-butyl ketone.

OR

(a) Predict the products of the following reactions:

Page 5 of 10
(i)

(i) Cl2 / Red phosphorous


CH3-CH2-COOH
(ii) H2O
(ii)
CH3

(i) CS2
+ CrO2Cl2
(ii) H3O+

(b) Arrange the following compounds in increasing order of acid strength:


Benzoic acid, 4-Nitrobenzoic acid, 4-Methoxybenzoic acid.
21.
(i) Identify the monomer in the following polymeric structure:
CN

[ CH2-CH=CH-CH2-CH2-CH ]n

(ii) On the basis of forces between their molecules in a polymer to which class does

neoprene belong?

(iii) Can both addition and condensation polymerization result in the formation of a co-

polymer?

22.
(i) Which of the following biomolecule is insoluble in water? Justify.
Insulin, Haemoglobin, Keratin.
(ii) Draw the Haworth structure for α-D-Glucopyranose.
(iii) Write chemical reaction to show that glucose contains aldehyde as carbonyl group.

23. John had gone with his mother to the doctor as he was down with fever. He then went to the
chemist shop with his mother to purchase medicines prescribed by the doctor. There he
observed a young man pleading with the chemist to give him medicines as he had nasal
congestion. The chemist gave him cimetidine. John advised and also explained to the young
man that he should only take the medicines prescribed by the doctor.

Answer the following questions:


a) Did the chemist give an appropriate medicine? Justify your answer.

Page 6 of 10
b) John’s action was appreciated by his mother. List any two reasons.
24. (a) Write the mechanism of hydration of ethene to form ethanol.
(b) How are the following conversions carried out?
(i) Propanol to propan-2-ol.
(ii) Propanol to 1-propoxypropane.
(c) Give the structure and the IUPAC name of the major product obtained in the following
reaction:
OH

conc. HNO3

OR
(a) Write the mechanism of the reaction of HI with methoxymethane.
(b) Identify A and B in the following reactions:
(i)
OH

NaOH (i) CO2


A B
(ii) H+

(ii)

Cu, 573 K CH3MgBr


C2H5OH A B
H2O / H+

(c) Give the structure and the IUPAC name of the major product obtained in the following
reaction:

OC2H5 conc. HNO3

conc. H2SO4

25. (a) A blackish brown coloured solid (A) which is an oxide of manganese, when fused with
alkali metal hydroxide and an oxidizing agent like KNO3, produces a dark green coloured

Page 7 of 10
compound (B). Compound (B) on disproportionation in neutral and acidic solution gives a
purple coloured compound (C). Identify A, B and C and write the reaction involved when
compound (C) is heated to 513 K.
(b)

(i) E0 M3+ / M2+values for the first series of transitionelements are given below.
Answer the question that follows:

E0 (V) Ti V Cr Mn Fe Co
M3+ / M2+ -0.37 -0.26 -0.41 +1.57 +0.77 +1.97

Identify the two strongest oxidizing agents in the aqueous solution from the
above data.

ii) Copper (I) ion is not known in aqueous solution


iii) The highest oxidation state of a metal is exhibited in its oxide.
OR
(a) Write balanced equations to represent what happens when
(i) Cu2+ is treated with KI.
(ii) Acidified potassium dichromate solution is reacted with iron (II) solution.
(ionic equation)
(b)
i) The figure given below illustrates the first ionization enthalpies of first, second
and third series of transition elements. Answer the question that follows

Page 8 of 10
Which series amongst the first, second and third series of transition elements have
the highest first ionization enthalpy and why?
ii) Separation of lanthanide elements is difficult. Explain.
iii) Sm2+, Eu2+ and Yb2+ ions in solutions are good reducing agents but an
aqueous solution of Ce4+ is a good oxidizing agent. Why?
26.
i) Graphically explain the effect of temperature on the rate constant of reaction? How can
this temperature effect on rate constant be represented quantitatively?
ii) The decomposition of a hydrocarbon follows the equation
−28000K
k = (4.5 x 10 11 s −1 ) e T

Calculate Ea
OR
i) In the reaction
Q + R → Products
The time taken for 99% reaction of Q is twice the time taken for 90% reaction of Q.
The concentration of R varies with time as shown in the figure below:

What is the overall order of the reaction? Give the units of the rate constant for the
same. Write the rate expression for the above reaction.

ii) Rate constant for a first order reaction has been found to be 2.54 x 10-3s-1.
Calculate its three-fourth life.

Page 9 of 10
Page 10 of 10
SAMPLE QUESTION PAPER
CLASS XII
CHEMISTRY

Time: 3 hrs MM: 70

GENERAL INSTRUCTIONS:
1. All the questions are compulsory.
2. Q. No. 1 to 5 are very short answer type, carrying 1mark each.
3. Q. No. 6 to 10 are short answer type, carrying 2 marks each.
4. Q. No. 11 to 22 are short answer type, carrying 3 marks each.
5. Q.No.23 is a value based question carrying 4 marks.
6. Q. No. 24 to 26 are long answer type, carrying 5 marks each.
7. Use of calculators is not allowed, use log tables wherever required.

1. Name the non stoichiometric point defect responsible for colour in alkali metal halides.

2. What is shape selective catalysis?

3. Amongst the isomeric alkanes of molecular formula C5H12, identify the one that on
photochemical chlorination yields a single monochloride.

4. Give the IUPAC name and structure of the amine obtained when 3-chlorobutanamide
undergoes Hoffmann –bromamide reaction.

5. How many ions are produced from the complex,[Co (NH3)6]Cl2 in solution?

6. Will the elevation in boiling point be same if 0.1 mol of Sodium chloride or 0.1 mol of
sugar is dissolved in 1L of water? Explain.

7. The following curve is obtained when molar conductivity (  m ) is plotted against the
square root of concentration, c1/2 for two electrolytes A and B

(a) How do you account for the increase in the molar conductivity of the electrolyte A
on dilution.
(b) As seen from the graph, the value of limiting molar conductivity ( o m ) for
electrolyte B cannot be obtained graphically. How can this value be obtained?

8. Name the following:


(a) A transition metal which does not exhibit variation in oxidation state in its
compounds.
(b) A compound where the transition metal is in the +7 oxidation state.
(c) A member of the lanthanoid series which is well known to exhibit +4 oxidation
state.
(d) Ore used in the preparation of Potassium dichromate.

9. Arrange the following in order of property indicated for each set:


(a) F2, Cl2, Br2, I2 – increasing bond dissociation enthalpy
(b) PH3, AsH3, BiH3, SbH3, NH3 – increasing base strength

10. (a) Predict the major product of acid catalysed dehydration of 1-Methylcyclohexanol.
(b) You are given benzene, conc.H2SO4, NaOH and dil.HCl. Write the preparation of
phenol using these reagents.

OR
Draw the structures of any two isomeric alcohols (other than 1o alcohols) having
molecular formula C5H12O and give their IUPAC names.

11. An element occurs in the bcc structure with cell edge of 288 pm. The density of the
element is 7.2 g cm-3. How many atoms of the element does 208g of the element
contain?

12. Calculate the boiling point of a 1M aqueous solution (density 1.04 g mL-1) of Potassium
chloride (Kb for water = 0.52 K kg mol-1, Atomic masses: K=39u, Cl=39.9u)
Assume, Potassium chloride is completely dissociated in solution

13. A galvanic cell consists of a metallic zinc plate immersed in 0.1M Zn(NO3)2 solution
and metallic plate of lead in 0.02M Pb(NO3)2 solution. Calculate the emf of the cell.
Write the chemical equation for the electrode reactions and represent the cell.
(Given: E o Zn 2 / Zn  0.76V ; E o Pb2 / Pb  0.13V )

14. Answer the following questions:


(a) What happens when a freshly precipitated Fe (OH)3 is shaken with a little amount
of dilute solution of FeCl3?
(b) Why are lyophilic colloidal sols more stable than lyophobic colloidal sols?
(c) What form Freundlich adsorption equation will take at high pressure?
15. What chemical principle is involved in choosing a reducing agent for getting the metal
from its oxide ore? Consider the metal oxides, Al2O3 and FeO and justify the choice of
reducing agent in each case.

OR

Account for the following facts:


(a) the reduction of a metal oxide is easier if the metal formed is in the liquid state at
the temperature of reduction.
(b) Limestone is used in the manufacture of pig iron from haematite.
(c) Pine oil is used in the froth floatation process used to concentrate sulphide ores.

16. (i) For M2+/M and M3+/M2+ systems, Eo values for some metals are as follows:
Cr2+/Cr = −0.9V Cr3+/Cr2+ = −0.4V
Mn2+/Mn = −1.2V Mn3+/Mn2+ = +1.5V
2+
Fe /Fe = −0.4V Fe3+/Fe2+ = +0.8V

Use this data to comment upon


(a) the stability of Fe3+ in acid solution as compared to that of Cr3+ and Mn3+
(b) the ease with which iron can be oxidised as compared to the similar process
for either Cr or Mn metals
(ii) What can be inferred from the magnetic moment of the complex K4[Mn(CN)6]
Magnetic moment: 2.2 BM?

17. (i) Describe the type of hybridisation for the complex ion [Fe(H2O)6]2+.
(ii) Write the IUPAC name of the ionisation isomer of the coordination compound
[Co(NH3)5Br]SO4. Give one chemical test to distinguish between the two
compounds.
18. (a) Explain why the dipole moment of chlorobenzene is lower than that of cyclohexyl
chloride.
(b) An optically active compound having molecular formula C7H15Br reacts with
aqueous KOH to give a racemic mixture of products. Write the mechanism involved in
this reaction.

19. Give the structures of A, B and C in the following reactions:

(a) C6 H 5 NO2 Fe


/ HCl
 A HNO
  B C
2 ; 273 K
 C
6 H 5OH


3 ;
(b) C6 H 5 N2Cl CuCN
 A H   B NH
2O / H
 C

20. (a) A non reducing disaccharide ‘A’ on hydrolysis with dilute acid gives an equimolar
mixture of D-(+)-glucose and D-(-)-Fructose.

A  H 2O HCl

 C6 H12O6  C6 H12O6
[ ]D = +66.50 + 52.5o −92.4o
Identify A. What is the mixture of D-(+)- glucose and D-(-)-Fructose known as?
Name the linkage that holds the two units in the disaccharide.

(b)  -amino acids have relatively higher melting points than the corresponding halo
acids. Explain.

21. (a) Pick out the odd one from among the following on the basis of their medicinal
properties mentioning the reason: Luminal, Seconal, Phenacetin, Equanil.

(b) Give an example of a substance that can act as a disinfectant as well as antiseptic
depending upon its concentration. (Specify concentration)

(c) Name any two macromolecules chosen as drug targets.

22. The following is not an appropriate reaction for the preparation of tert.-butyl ethyl
ether:
C2 H 5ONa  (CH 3 )3 C  Cl  (CH 3 )3 C  OC2 H 5
(i) What would be the major product of the given reaction?
(ii) Write a suitable reaction for the preparation of tert.-butyl ethyl ether,
specifying the names of reagents used.
Justify your answer in both cases.

23. Study the given passage carefully and answer the questions that follow:

Shalini studied a chapter on Polymers in school and came across the following
paragraph:
The durability, strength, low cost, water and chemicals resistance, welding properties,
lesser energy, fewer atmosphere emissions and light weight are advantages of plastic
bags.

Shalini is confused as she has been reading in the newspaper about the ban on the usage
of plastic substances.

She further finds that despite the durability, the use of these materials has presented
mankind with serious waste disposal problem as these materials do not disintegrate by
themselves. In view of this, certain polymers are being developed which are broken
down rapidly by microorganisms. Shalini feels relaxed that such kinds of biomaterials
are being developed.

(a) Name the class of these useful polymers which do not harm the environment.
(b) Give any one example of these polymers and name its monomers.
(c) Comment on the qualities of Shalini.

24. (a) Give a plausible explanation for each one of the following:
(i) Although phenoxide ion has more number of resonating structures than
carboxylate ion, carboxylic acid is a stronger acid than phenol.
(ii) There are two -NH2 groups in semicarbazide. However, only one is in
volved in the formation of semicarbazones.
(b) Carry out the following conversions in not more than two steps:
(i) Phenyl magnesium bromide to benzoic acid.
(ii) Acetaldehyde to But-2-enal.
(iii) Benzene to m-Nitroacetophenone

OR

(a) Give a simple chemical test to distinguish between the pair of organic compounds:
Ethanal and Propanal
(b) Name and complete the following chemical reaction:
RCH 2COOH (  
i ) X 2 / redP( ii ) H 2O

(c) Draw the structures of the following derivatives:


(i) The 2,4-Dinitrophenylhydrazone of benzaldehyde
(ii) Acetaldehydedimethyl acetal
(iii) Cyclopropanone oxime

25. (a) Write the rate law for a first order reaction. Justify the statement that half life for a
first order reaction is independent of the initial concentration of the reactant.

(b) For a first order reaction, show that the time required for 99% completion of a
first order reaction is twice the time required for the completion of 90%.
OR
(a) For the reaction A→B, the rate of reaction becomes twenty seven times when the
concentration of A is increased three times. What is the order of the reaction?

(b) The activation energy of a reaction is 75.2 kJmol-1 in the absence of a catalyst
and it lowers to 50.14 kJmol-1 with a catalyst. How many times will the rate of
reaction grow in the presence of a catalyst if the reaction proceeds at 25oC?

26. (a) Write balanced chemical equations for the following:


(i) Complete hydrolysis of XeF6.
(ii) Disproportionation reaction of orthophosphorus acid.
(b) Draw the structure of a noble gas species which is isostructural with BrO3−.
(c) Considering the parameters such as bond dissociation enthalpy, electron gain
enthalpy and hydration enthalpy, compare the oxidising power of F2 and Cl2.
(d) Why is K a2  K a1 for H2SO4 in water?

OR
Explain the following:
(a) Hydrogen fluoride is a weaker acid than hydrogen chloride in aqueous solution..
(b) PCl5 is ionic in nature in the solid state.
(c) SF6 is inert towards hydrolysis.
(d) H3PO3 is diprotic.
(e) Out of noble gases only Xenon is known to form established chemical compounds.
SCORING KEY
SAMPLE PAPER (CHEMISTRY)
CLASS XII

Q.No Answers Marks


1. Metal excess or anionic vacancies or F-centres 1

2. Catalysis by zeolites which depends on the shape and size of the reactants and 1
the products as compared to those of the pores and cavitites of zeolites.
3. Neopentane or 2,2-Dimethylpropane 1

4. 3-Chloropropanamine, CH3CH(Cl)CH2NH2 ½, ½

5. Three ions [Co(NH3)6]2+, 2Cl− 1

6. No, the elevation in boiling point is not the same. ½


Elevation in boiling point is a colligative property which depends on the 1½
number of particles. NaCl is an ionic compound which dissociates in solution
to give more number of particles whereas sugar is made up of molecules and
thus does not dissociate.

7. (a) As seen from the graph, electrolyte A is a strong electrolyte which is 1


completely ionised in solution. With dilution, the ions are far apart from each
other and hence the molar conductivity increases.
(b) To determine the value of limiting molar conductivity for electrolyte B, 1
indirect method based upon Kohlrausch law of independent migration of ions
is used.
8. (a) Scandium (Sc) ½
(b) KMnO4 or any other suitable example ½
(c) Cerium (Ce) or any other example. ½
(d) Chromite ore ½

9. (a) I2 < F2 < Br2 < Cl2 1


(b) BiH3 < SbH3 < AsH3 < PH3 < NH3 1
10. (a) 1-Methylcyclohexene 1
(b)
. H 2 SO4 , 
C6 H 6 conc
  C6 H 5 SO3 H NaOH
 , fuse
,575

 K
 C6 H 5ONa dil
. HCl
 C6 H 5OH 1

OR
Any two isomers out of the following:
(i) CH3−CH2−CH2−CH(OH)− CH3 Pentan-2-ol ½+½
(ii) CH3−CH2−CH(OH)- CH2− CH3 Pentan-3-ol
(iii) CH3−CH(CH3) −CH(OH) −CH3 3-Methylbutan-2-ol ½+½
(iv) CH3−CH2−C(CH3)(OH) − CH3 2-Methylbutan-2-ol
11. For the bcc structure, z = 2
Z M
Density   3 ½
a  No
2 M
7.2 g cm 3  10
(288 10 cm)  (6.022 10 23 )mol 1
3
½
Or M = 51.8 g mol-1 ½
By mole concept,
51.8 g of the element contains 6.022 × 1023 atoms
6.022 1023  208 ½
208 g of the element will contain atoms
51.8
= 24.17 × 1023 atoms. ½

12. Molar mass of KCl = 39+35.5 = 74.5 g mol-1


As KCl dissociates completely, number of ions produced are 2.
Therefore, van’t Hoff factor, i=2 ½
Mass of KCl solution = 1000 × 1.04 = 1040 g
Mass of solvent = 1040 – 74.5 = 965.5 g = 0.9655 kg
Molality of the solution:
½
No. of moles of solute 1mol
  1.0357 m
Mass of solvent in kg 0.9655 kg
½
Tb  i  K b  m
 2  0.52 1.0357  1.078o C ½
Therefore, boiling point of solution = 100 + 1.078 = 101.078 oC 1
(deduct ½ mark if unit not written/incorrect)

13. Anode reaction: Zn( s)  Zn 2 (aq)  2e  ½


½
Cathode reaction: Pb2 (aq)  2e   Pb( s)
½
Cell representation: Zn( s) / Zn 2 (aq) // Pb2 (aq) / Pb( s)
According to Nernst equation:
0.059 [ Zn 2 ]
Ecell  E o cell  log ½
n [ Pb2 ]
0.059 0.1
Ecell  [0.13  (0.76)]  log ½
2 0.02
 0.63  0.02955  log 5
 0.63  0.02955  0.6990
 0.63  0.0206  0.6094V ½

14. (a) A reddish brown coloured colloidal solution is obtained. 1


(b) Stability of lyophilic sols is due to:
(i) same charge on all the colloidal particles. ½+½
(ii) solvation of the colloidal particles.
(c) At high pressures, amount of gas adsorbed (x/m) becomes independent of
pressure (P) 1
x
 k  Po
m
15. The feasibility of thermal reduction can be predicted on the basis of 1
Ellingham diagram. Metals for which the standard free energy of formation (
 f Go ) is more negative can reduce those metals for which  f Go is less
negative. At a given temperature, any metal will reduce the oxide of other
metals which lie above it in the Ellingham diagram.
(a) Below the temperature approx 1623K), corresponding to the point of
intersection of Al2O3 and MgO curves, Mg can reduce alumina. 1
(b) At temperatures below 1073K, the CO,CO2 line lies Fe, FeO line, thus CO
is a better reducing agent.
At temperatures above 1073K, Coke will reduce FeO and itself get oxidised
to CO. 1
OR

(a) Entropy is higher when a metal is in the liquid state than when it is in the
1
solid state. Thus TS increases, thus G o becomes more negative and the
reduction becomes easier.( G  H  TS )
(b) Limestone provides the flux (CaO) which combines with the impurities 1
(SiO2) to form slag (CaSiO3). Thus it helps in the removal of impurities.
(c) Pine oil (Collector) enhances the non wettability of the ore particles,
which become lighter and hence rise to the surface along with the froth. 1
3+ 2+ 3+
16. (i)(a) Cr /Cr has a negative reduction potential. Hence Cr cannot be
reduced to Cr2+. Cr3+ is most stable. Mn3+/Mn2+ have large positive Eo values. 1
Hence Mn3+ can be easily reduced to Mn2+. Thus Mn3+ is least stable.
Fe3+/Fe2+ couple has a positive Eo value but small. Thus the stability of Fe3+ is
more than Mn3+ but less stable than Cr3+.
(b) If we compare the reduction potential values, Mn2+/Mn has the most
negative value i.e its oxidation potential value is most positive. Thus its most 1
easily oxidised. Thus the decreasing order for their ease of oxidation is Mn >
Cr >Fe.
(ii) K4[Mn(CN)6]
Mn is in +2 oxidation state. Magnetic moment 2.2 indicates that it has one
unpaired electron and hence forms inner orbital or low spin complex. In 1
presence of CN− is a strong ligand, hybridisation involved is d2sp3 (octahedral
complex)

17. (i) Fe exists as Fe2+. There are 4 unpaired electrons. Water is a weak ligand.
Thus the hybridisation involved is sp3d2. It is an octahedral outer orbital 1
complex.
(marks to be granted if hybridisation is depicted diagrammatically)
(ii) The ionisation isomer is [Co(NH3)5SO4]Br. The IUPAC name is 1
Pentaamminesulphatocobalt(III)bromide.
The isomer [Co(NH3)5 Br] SO4 gives a white precipitate of BaSO4 with BaCl2 1
solution whereas the isomer [Co(NH3)5 SO4]Br does not form this precipitate.
(or any other relevant test)
18. (a) Due to greater s-character, a sp2 hybrid carbon is more electronegative
than a sp3 hybrid carbon. Therefore, the sp2 hybrid carbon of C-Cl bond in
chlorobenzene has less tendency to release electrons to Cl than a sp3 hybrid 1
carbon of cyclohexyl chloride.
( marks to be granted if shown with the help of a figure)

(b) Since the alkyl halide reacts with KOH to form a racemic mixture, it must ½
be a 3o alkyl halide and the reaction will follow SN1 mechanism.

19. (a) A – C6H5NH2 B – C6H5N2+Cl− C – C6H5-N2 –C6H4 -OH ½+½+½


(b) A - C6H5CN B - C6H5COOH C - C6H5CONH2 ½+½+½

20. (a) A – Sucrose (C12H22O11) ½


The mixture of D-(+)- glucose and D-(-)-Fructose is known as invert sugar. ½
The linkage which holds the two monosaccharide units through oxygen atom
is called glycosidic linkage. 1
(b) The amino acids exist as dipolar zwitter ion. Due to this dipolar salt like
character they have strong dipole dipole attractions Thus their melting points 1
are higher than the corresponding haloacids which do not exist as zwitter ions.
21. (a) Phenacetin is an antipyretic, while the rest are tranquilizers. ½+½
(b) 0.2% solution of phenol acts as antiseptic whereas 1% solution of phenol 1
acts as disinfectant.
(c) Carbohydrates, proteins, nucleic acids, lipids (any two) ½+½

22. (i) Since the alkyl halide is a 3o halide and C2H5ONa is a strong base,
therefore elimination occurs preferably. The product obtained is 2- 1
Methylprop-1-ene. CH3−C(CH3)=CH2
(ii) To prepare t-Butyl ethyl ether, the alkyl halide should be 1o i.e. ½+½
chloroethane and the nucleophile should be sodium t-butoxide because the 3o
nucleophile is able to attack 1o alkyl halide. 1
(CH 3 )3 C  O  Na   CH 3CH 2  Cl  (CH 3 )3 C  OCH 2CH 3

23. (a) The class of polymers is Biodegradable polymers. 1


(b) One example of biodegradable polymers is PHBV (Poly-  - 1
hydroxybutyrate-co-  -hydroxyvalerate).
The names of its monomers are: 3-hydroxybutanoic acid and 3- 1
hydroxypentanoic acid
(c) Care for environment, concern for the health of the people or any other
two relevant points. ½+½

24. (a) (i) The delocalisation of benzene electrons contributes little towards the 1
stability of phenoxide ion. The carboxylate ion is much more
resonance stabilized than phenoxide ion.
(ii) Semicarbazide has two –NH2 groups. One of them, which is directly 1
attached to C=O is involved in resonance. Thus electron density on
this group decreases and it does not act as a nucleophile. In contrast,
the lone pair of electrons on the other –NH2 group is available for
nucleophilic attack.

(b) (i) PhMgBr + O=C=O → PhCOOMgBr H



2O
PhCOOH 1

(ii)
 1
2CH 3CHO OH  CH 3CH (OH )  CH 2CHO heat CH 3CH  CH  CHO
(iii)
. H 2 SO4  conc. HNO3
C6 H 6 (   PhCOCH 3 conc
CH 3CO ) 2 O , anhAlCl 3
     m  NO2 C6 H 4 COCH 3 1
or CH3COCl/AlCl3
OR
(a) Ethanal and propanal can be distinguished by Iodoform test.
Ethanal gives a yellow precipitate of iodoform with an alkaline solution of 1
NaOH. Propanal does not give this test.

CH 3CHO  4 NaOH  3I 2  CHI 3  HCOONa  3H 2O  3NaI

½
(b) RCH 2 COOH (   RCH ( X )COOH
i ) X 2 ,red P ( ii ) H 2O

The name of the reaction is Hell Vohlard Zelinsky reaction ½

(c) (i)
1

1
(ii)

(iii)
1

25. (a) For a first order reaction 1


2.303 [ R]o
k log where [R]o = initial concentration, [R] = conc. after
t [ R]
time t
When half of the reaction is completed, [R] = [R]o/2. Representing, the time
taken for half of the reaction to be completed, by t1/2, equation becomes:

2.303 [ R]o
k log ½
t1/ 2 [ R]o / 2

2.303
t1/ 2  log 2
k
2.303
t1/ 2   0.3010
k
½
0.693
t1/ 2 
k
The above equation shows that half life first order reaction is independent of
the initial concentration of the reactant.

(b) For a first order reaction


2.303 a 1
t log
k ax
2.303 a
t99%  log
k a  0.99a
2.303 a ½
t90%  log
k a  0.90a ½
t99%  2  2.303  2.303
 / =2
t90%  k  k
t99%  2  t90% 1
OR

(a) r = k[R]n
½
When concentration is increased three times, [R] = 3a
27r = k(3a)n
27r k (3a) n
 n
or 27 = 3n or 33 = 3n ½
r ka
n=3
1
(b)
According to Arrhenius equation,
Ea
log k  log A 
2.303RT
For uncatalysed reaction
Ea (1) ½
(i) log k1  log A 
2.303RT
For catalysed reaction
E a ( 2) ½
(ii) log k 2  log A 
2.303 RT
A is equal for both the reactions.
Subtracting equation(i) from equation(ii)
k E (1)  Ea (2)
log 2  a
k1 2.303RT
k2 (75.2  50.14)kJ mol 1 1
log 
k1 2.303  8.314 JK 1mol 1  298 K
k
log 2  4.39
k1
k2
 anti log( 4.39) 1
k1
= 2.45×104
Rate of reaction increases by 2.45×104 times.

26. (a) (i) XeF6 ( s)  3H 2O(l )  XeO3 ( s)  6 HF (aq) 1


(ii) 4 H 3 PO3 
 PH 3  3H 3 PO4
heat
1
(b) XeO3 is isostructural with BrO3−. (pyramidal structure)

(c) The bond dissociation enthalpy of F-F bond is lower than that of Cl-Cl
bond and hydration enthalpy of F− ion is much higher than that of Cl− ion. 1
These two factors more than compensate the less negative electron gain
enthalpy of F2. Thus, F2 is a stronger oxidizing agent than Cl2.

(d) H2SO4 ionises in two stages and hence has two dissociation constants. 1
K a2  K a1
.
This is because the negatively charged HSO4− ion has much less tendency to
donate a proton to H2O as compared to neutral H2SO4.

OR

(a) Due to stronger H-F bond than HCl bond, HF ionises less readily than HCl
in aqueous solution to give H+ ions. Therefore HF is a weaker acid than HCl. 1
(b) In solid state, PCl5 consists of ions [PCl4]+[PCl6]− . On melting these ions
become free to move and hence conducts electricity in the molten state. 1
(c) In SF6, S is sterically protected by six F atoms and hence does not allow
H2O molecules to attack the S molecule. Also, F does not have d-orbitals to 1
accept the electrons donated by H2O molecules.

(d) In the structure of H3PO3 , it contains only two ionisable H-atoms which
are present as -OH groups, thus it behaves as a dibasic acid. 1
(e) Except radon which is radioactive, Xenon has least ionisation energy
among noble gases and hence it readily forms chemical compounds
particularly with O2 and F2. 1
CBSE SAMPLE PAPER -2016-17
NAME OF THE UNIT TOTAL VSA SA SA VBQ LA WEIG
WEIGH (1 mark) (2 marks) (3 marks) (4marks (5 marks) HTA
TAGE ) GE
Solid State 23 1 (1) 3 (1) 4 (2)
Rememberi Understanding
ng
Solutions 2 (1) 3 (1) 5 (2)
Rememberi Evaluation
ng
Electrochemistry 5 (1) 5 (1)
Application

Chemical Kinetics 2 (1) 3 (1) 5 (2)


Application Application

Surface Chemistry 1 (1) 3 (1) 4 (2)


Rememberi Evaluation
ng
General Principles and 19 3 (1) 3 (1)
Processes of Isolation of Application
Elements
p - Block Elements 2 (1) 3 (1) 5 (2)
Understand Application
ing
d - and f - Block 1 (1) 5 (1) 6 (2)
Elements HOTS Understanding
Coordination 2 (1) 3 (1) 5 (2)
Compounds Application HOTS
Haloalkanes and 28 1 (1) 3 (1) 4 (2)
Haloarenes Evaluation Understanding
Alcohols, Phenols and 1 (1) 3 (1) 4 (2)
Ethers HOTS understanding
Aldehydes, Ketones and 5 (1) 5 (1)
Carboxylic acids HOTS

Amines 2 (1) 3 (1) 5 (2)


Understand Application
ing
Biomolecules 3 (1) 3 (1)
Remembering
Polymers 3 (1) 3 (1)
Understanding
Chemistry in Everyday 4 (1)* 4 (1)
Life Value
based
question
TOTAL 5 (5) 10 (5) 36 (12) 4 (1) 15 (3) 70
(26)

* Value based question


A(B) WHERE; A = NUMBER OF QUESTIONS; B= MARKS
HENCE 26 (70)
Sample Question Paper Chemistry class XII 2016-17

MM:70 TIME 3 HRS

1. Define Kraft temperature. 1


2. The electronic configuration of a transition element in +3 oxidation state is 1
[Ar]3d7. Find out its atomic number.
3. Draw the structure of 4-tertbutyl-3-iodoheptane. 1

4. Give the equation of reaction for the preparation of phenol from cumene. 1
5. Name the type of semiconductor obtained when silicon is doped with boron. 1

6. The two complexes of nickel, [Ni(CN)4]2- and [Ni(CO)4], have different 2


structures but possess same magnetic behaviour. Explain.
OR
A chloride of fourth group cation in qualitative analysis gives a green coloured
complex [A] in aqueous solution which when treated with ethane –1, 2 –
diamine (en) gives pale - yellow solution [B] which on subsequent addition of
ethane –1, 2 – diamine turns to blue/purple [C] and finally to violet [D]. Write
the structures of complexes [A], [B], [C] and [D].

7. Account for the following : 2


(i) XeF2 is linear molecule without a bend.
(ii) The electron gain enthalpy with negative sign for fluorine is less than that of
chlorine, still fluorine is a stronger oxidizing agent than chlorine.

8. Derive the relationship between relative lowering of vapour pressure and mole 2
fraction of the volatile liquid.

9. After 24 hrs, only 0.125 gm out of the initial quantity of 1 gm of a radioactive 2


isotope remains behind. What is its half life period?

10. Write the IUPAC names of the following: 2

11. The edge length of a unit cell of a metal having molecular mass 75 g/mol is 5 A◦ 3
which crystallises in a cubic lattice. If the density is 2g/cc, then find the radius
of the metal atom.

12. (i) A mixture of X and Y was loaded in the column of silica. It was eluted by 3
alcohol water mixture. Compound Y eluted in preference to compound X.
Compare the extent of adsorption of X and Y on column.
(ii) Why copper matte is put in silica lined converter? Write reactions involved
(iii)Name the method used for the refining of Zr.
13. (i) Complete the following chemical equations. 3
(a) NH4Cl (aq.)+ NaNO2 (aq. ) →
(b) P4 + 3NaOH + 3H2O→
(ii) Why is Ka2 << Ka1 for H2SO4 in water?

14. Write the correct formulae for the following coordination compounds: 3
(i) CrCl3.6H2O (violet with 3 chloride ions precipitated as AgCl)
(ii) CrCl3.6H2O (light green colour, with 2 chloride ions precipitated as AgCl )
(iii)CrCl3.6H2O (dark green colour, with 1 chloride ion precipitated as AgCl )

15. Give reasons for the following observations: 3


(i) p-dichlorobenzene has higher melting point than those of o and m –isomers.
(ii) Haloarenes are less reactive than haloalkanes towards nucleophillic
substitution reaction.
(iii) The treatment of alkyl chloride with aqueous KOH leads to the formation of
alcohol but in the presence of alcoholic KOH, alkene is the major product,

16. (i) Why does leather get hardened after tanning? 3


(ii) On the basis of Hardy-Schulze rule explain why the coagulating power of
phosphate is higher than chloride.
(iii)Do the vital functions of the body such as digestion get affected during
fever? Explain your answer.

17. Calculate the mass of a non-volatile solute (molar mass40 g/mol) which should 3
be dissolved in 114 g octane to reduce its vapour pressure to 80%.
OR
At 300 K, 36 g of glucose, C6H12O6 present per litre in its solution has an
osmotic pressure of 4.98 bar. If the osmotic pressure of another glucose solution
is 1.52 bar at the same temperature, calculate the concentration of the other
solution.

18. Carry out the following conversions : 3


i) Phenol to benzoquinone.
ii) Propanone to 2-Methylpropan-2-ol.
iii) Propene to propan-2-ol.

19. (i) Illustrate the following reactions: 3


a) Hoffmann bromamide degradation reaction.
b) Coupling reaction.

(ii) Write a chemical test to distinguish between aniline and methylamine.

20. (i) Name the common types of secondary structure of proteins and give one 3
point of difference.
(ii) Give one structural difference between amylose and amylopectin

21. Observe the graph in diagram and answer the following questions. 3
(i) If slope is equal to -2.0x10-6 sec-1, what will be the value of rate constant?
(ii) How does the half-life of zero order reaction relate to its rate constant?

22. (i) Classify the following as addition and condensation polymers: Terylene, 3
Bakelite, Polyvinyl chloride, Polythene.
(ii) Explain the difference between Buna – N and Buna – S.

23. Ali’s brother likes taking medicines. He sometimes even takes cough syrups 4
even when he is not ill. One such day, he took cough syrup when he was
healthy. After some time he started feeling nausea, headache and his body
started itching. Ali’s father did not take him to the doctor and wanted to give
medicine on his own. Ali insisted that his father should not give medicine to his
brother on his own but should take him to a doctor.

After reading the above passage, answer the following questions:


(i) Mention the values shown by Ali.
(ii) Why did his body start itching and what kind of medicine will doctor
prescribe him?
(iii) Why medicines should not be taken without consulting doctor?
(iv) Give one point of difference between agonist and antagonist.

24. (i) State the relationship amongst cell constant of a cell, resistance of the 5
solution in the cell and conductivity of the solution. How is molar
conductivity of a solution related to conductivity of its solution?
(ii) A voltaic cell is set up at 25◦C with the following half cell;
Al / Al 3+ (0.001 M) and Ni /Ni 2+ (0.50 M)
Calculate the cell voltage. [ E◦ Ni2+/Ni = - 0.25V, E◦Al3+/Al = -1.66V]
OR
(i) Calculate the potential of hydrogen electrode in contact with a solution
whose pH is 10.
(ii) State Faraday’s laws of electrolysis. How much charge in terms of
Faraday is required for reduction of 1 mol of Cr 2O72- to Cr 3+ ?

25. (i) Is the variability in oxidation number of transition elements different from 5
that of non – transition elements? Illustrate with examples.
(ii) Give reasons:
(a) d- block elements exhibit more oxidation states than f-block elements.
(b) Orange solution of potassium dichromate turns yellow on adding
sodium hydroxide to it.
(c) Zirconium (Z= 40) and Hafnium (Z = 72) have almost similar atomic
radii.
OR
(i) Describe the preparation of potassium permanganate from pyrolusite
ore. Write balanced chemical equation for one reaction to show the
oxidizing nature of potassium permanganate.
(ii) Draw the structures of chromate and dichromate ions.

26. (i) A ketone A which undergoes haloform reaction gives compound B on 5


reduction. B on heating with sulphuric acid gives compound C, which
forms mono-ozonide D. The compound D on hydrolysis in presence of
zinc dust gives only acetaldehyde. Write the structures and IUPAC names
of A, B and C. Write down the reactions involved.
(ii) Predict the products formed when cyclohexanecarbaldehyde reacts with
following reagents.
(a) PhMgBr and then H3O+.
(b) Tollens’ reagent.
OR
(i) Complete each synthesis by giving missing starting material, reagent or
products:
+
O H
(a) HO NH2
+

CH3 KMnO4
(b)
KOH, heat
O

OH SOCl2
(c)
O 

OH
O

CH2
H
(d)

(e)
MARKING SCHEME

1. The formation of micelle takes place only above a particular temperature called 1
Kraft temperature.
2. 28 1

3. 1

4. 1

5.
P type semiconductor 1

6. [Ni(CN)4]2- ½+1/2+
dsp2 hybridisation, Ni in +2 state 1/2+1/2
all electrons are paired, so diamagnetic.

[Ni(CO)4]
sp3 hybridisation, Ni in 0 state
all electrons are paired so diamagnetic

OR
2+
A=[Ni(H2O)6]
B=[Ni(H2O)4(en)]2+
C=[Ni(H2O)2(en)2]2+
D=[Ni(en)3]2+

7. (i) The electron arrangement is trigonal bipyramidal. The shape 1+1


is linear because the lone pairs prefer the equatorial positions.
The moleculeXeF2 has 3lone-pairs and 2bond-pairs.
(ii) Low bond dissociation enthalpy and high hydration enthalpy of flourine.
8. Let us assume a binary solution in which the mole fraction of the solvent be
x1 and that of the solute be x2, p1 be the vapour pressure of the solvent and
p1o be the vapour pressure of the solvent in pure state.
According to Raoult’s Law:
p1=x1p1o…………………………..(1) ½
The decrease in vapour pressure of the solvent (∆p1) is given by:

=> ∆p1=p1o-p1
½
=> ∆p1=p1o-p1ox1 [using equation (1)]

=> ∆p1=p1o (1-x1)

Since we have assumed the solution to be binary solution, x2=1-x1


½
=> ∆p1=p1ox2

=> x2= ∆p1/p1o ½

9. a=1g, a-x= 0.125g, t=24hours

½
k= 𝑡 log𝑎−𝑥
2.303 𝑎

½
k= 𝑡 log0.125
2.303 1

=0.0866hr-1.

½
t1/2=
0.693
𝑘

½
t1/2=0.0866
0.693

=8hours

10. (i) 1-Phenylmethanamine. 1

(ii) N,N-Dimethylmethanamine. 1
11.
𝑍𝑋𝑀 ½
ρ=
𝑎3 𝑋𝑁𝑎

2 x (5 x10 8 ) 3 x 6 x10 23
Z ½
75
1
=2

3
r=√ 𝑎 ½
4
3
r=√ 𝑥5
4

½
=2.165A0

12. (i) X is more strongly adsorbed than Y. 1


(ii) Copper matte contains small amount of FeO as impurity which is removed ½
as FeSiO3 slag when reacts with silica.
FeO + SiO2 -----> FeSiO3 (slag) ½
(iii) Van Arkel Method 1

13. (a) NH4Cl (aq.)+ NaNO2 (aq. ) →N2(g) +2H2O(l) +NaCl(aq.) 1


(b) P4 + 3NaOH + 3H2O→ 3NaH2PO2 + PH3
1

(iii) H2SO4 is a very strong acid in water because of its first ionisation to H 3O+. 1
and HSO4-. The ionization of HSO4- to H3O+ and SO42- is very small (it is
difficult to remove a proton from a negatively charged ion).
14. (i) [Cr(H2O)6]Cl3 1
(ii) [Cr(H2O)5Cl]Cl2H2O 1
(iii) [Cr(H2O)4(Cl)2]Cl(H2O)2 1
15. (i) It is due to the symmetry of para-isomers that fits in the crystal better as 1
compared to ortho and meta-isomers.
(ii) Resonance effect / Difference in hybridization of carbon atom in C-X bond / 1
Instability of phenyl cation / because of the repulsion, it is less likely for the
electron rich nucleophile to approach electron rich arenes .
(iii)Alkoxide ion present in alcoholic KOH, is not only a strong nucleophile but 1
also a strong base.
16. (i) Animal hides are colloidal in nature, havig positively charged particles, 1
when soaked in tannin, which contains negatively charged colloidal
particles, mutual coagulation occurs.
(ii) Greater the valency of flocculating ion added, greater is its power to cause 1
precipitation.
(iii)The optimum temperature range for enzymatic activity is 298-310 K i.e 1
enzymes are active beyond this temp. range, thus during fever the activity of
enzymes may be affected.
17 if vapour pressure of pure liquid is = Po
80 % of pure liquid Ps= 80×Po/100 = 0.8Po 1
Ps =Po × Xsolute
mass of solute = x gram ½
And mass of solvent = 114g
Molar mass of solute= 40 g/mol
Molar mass of solvent (octane C8H18) = 114g/mol
Number of moles of solute = x/40 = 0.025x
Number of moles of solvent = 114/114= 1 moles
Mole fraction of solvent = 1/(1+0.025x)
0.8Po=Po×1/(1+0.025x)
Cross multiply we get 1
(1+0.025x))0.8Po= Po
Divide by 0.8 Po we get
1+0.025x = 1.25
Subtract 1 both side we get
0.025x = 0.25
Now divide by 0.025 we get
x = 10g ½

OR
πV=CRT 1
4.98 = 36/180 x R x 300=60 R ..........(i) ½
1.52 = C x R x 300 ...........(ii) ½

Divide (ii) by (i)


C= 0.061M ½
½ for
unit
18.
1

(i)

(ii)

(iii)

19. 1

(i)
1

(ii)
(iii) Aniline will give azo dye test whereas methylamine will not 1
20. (i) α helix-Intramolecular H bonding. 1
β pleated-Intermolecular H bonding. 1
(ii) Amylose is a straight chain polymer of D glucose whereas amylopectin is a ½
branched polymer. ½

21. (i)
𝑘
Slope = - 2.303 1

k= - 2.303 x - 2.0x10-6 sec-1 1


= 4.606 x 10-6 sec-1.

A0 1
(ii) t1 
2 2k
22. (i) Addition polymers: Polyvinyl chloride, Polythene. ½+1/2
Condensation polymers: Terylene, Bakelite. ½+1/2

(ii) Buna- N: 1,3-Butadiene + Acrylonitrile. ½+1/2


Buna -S: 1,3-Butadiene + Styrene.
23. (i) Caring, empathetic, awareness, application of knowledge at right place. ½+1/2
(ii) Because of production of histamine. Doctor will prescribe antihistamine. ½+1/2
(iii)Medicines can be potent poisons. 1
(iv) An agonist is a chemical that binds to a receptor and activates the receptor ½+1/2
to produce a biological response.
Antagonist is a drug that blocks a receptor.
24. G* 1
(i) 
R
1000 
m  1
C
0.0591 1 1
E Ni 2 / Ni   0.25  log
2 0.50
= - 0.259V
0.0591 1
E Al 3 / Al  1.66  log
3 0.001 1
= - 1.719V

Ecell = 0.259V –(-1.719V) = 1.46V 1

OR

(i)
0.0591
EH+/1/2 H2 = E0H+/1/2 H2-
1
log[H+] 1
n
0.0591 1 ½
EH+/1/2 H2 = 0  log 10
1 10
= - 0.591V ½

(ii) First law-the chemical deposition due to flow of current through an 1


electrolyte is directly proportional to the quantity of electricity
(coulombs) passed through it.

Faraday's second law of electrolysis states that, when the same quantity of 1
electricity is passed through several electrolytes, the mass of the substances
deposited are proportional to their respective chemical equivalent or equivalent
weight.

3F 1

25. (i) In transition elements, the oxidation state differ by 1 e.g Cu+ and Cu2+. 1 +1
In non-transition elements, the oxidation state differ by 2 e.g Pb+2 and Pb4+
(ii)
(a) d- block elements exhibit more oxidation states because of comparable
energy gap between d and s subshell whereas f-block elements have 1
large energy gap between f and d subshell.
(b)
1

1
(c) Lanthanoid contraction.

OR
1
(i) 2 MnO2 + 4 KOH + O2 ----------> 2K2MnO4 + 2H2O

1
3 MnO42- + 4H+ ------------> 2MnO4- + MnO2 + 2H2O
1
MnO4 – + 5Fe2+ + 8H+ ⎯→ Mn2+ + 5Fe3+ + 4H2O

(ii)

1
26. 1

(ii)
(a)

(b)

OR

(a)

1
(b)
1

(c)
1

(d)

(e)
Chemistry (Theory)
Time Allowed: 3 hours Maximum Marks: 70
General Instructions:
(i) All questions are compulsory.
(ii) Questions number 1 to 5 are very short answer questions and carry 1 mark each.
(iii) Questions number 6 to 10 are short answer questions and carry 2 marks each.
(iv) Questions number 11 to 22 are also short answer questions and carry 3 marks each.
(v) Question number 23 is a value based question and carry 4 marks.
(vi) Questions number 24 to 26 are long answer questions and carry 5 marks each.
(vii) Use log tables, if necessary. Use of calculators is not allowed

1. (CH3)3C—CHO does not undergo aldol condensation. Comment. 1

2. In the process of wine making, ripened grapes are crushed so that sugar and 1
enzyme should come in contact with each other and fermentation should
start. What will happen if anaerobic conditions are not maintained during
this process?

3. A coordination compound with molecular formula CrCl3.4H2O precipitates 1


one mole of AgCl with AgNO3 solution. Its molar conductivity is found to be
equivalent to two ions. What is the structural formula and name of the
compound?

4. How is Brownian movement responsible for the stability of sols? 1

5.  Ea 1
In the Arrhenius equation, what does the factor e RT
corresponds to?

6. (i) Allyl cholride can be distinguished from Vinyl chloride by NaOH and silver 2
nitrate test. Comment.
(ii) Alkyl halide reacts with Lithium aluminium hydride to give alkane. Name
the attacking reagent which will bring out this change.

7. Which of the following solutions has higher freezing point? 2


0.05 M Al2(SO4)3 , 0.1 M K3[Fe(CN)6] Justify.

8. Calculate the emf of the following cell at 298 K : 2


Cr(s) / Cr3+ (0.1M) // Fe2+ (0.01M) / Fe(s)
0
[Given : ECell = + 0.30 V]
OR
The conductivity of 10 mol /L acetic acid at 250C is 4.1 x 10 -5 S cm-1.
-3

Calculate its degree of dissociation, if  0m for acetic acid at 250C is 390.5 S


cm2 mol-1.

9. What happens when: 2


(i) Orthophosphorus acid is heated?
(ii) XeF6 undergoes complete hydrolysis?

10. Identify the following: 2


(i) Oxoanion of chromium which is stable in acidic medium.
(ii) The lanthanoid element that exhibits +4 oxidation state.
(iii)
11. Give the IUPAC name of the product formed when: 3
(i) 2-Methyl-1-bromopropane is treated with sodium in the presence of
dry ether.
(ii) 1- Methyl cyclohexene is treated with HI.
(iii) Chloroethane is treated with silver nitrite.

12. The freezing point of benzene decreases by 2.12 K when 2.5 g of benzoic acid 3
(C6H5COOH) is dissolved in 25 g of benzene. If benzoic acid forms a dimer in
benzene, calculate the van’t Hoff factor and the percentage association of
benzoic acid. (Kf for benzene = 5.12 K kg mol-1)

13. Explain the following behaviours: 3


(i) Alcohols are more soluble in water than the hydrocarbons of
comparable molecular masses.
(ii) Ortho-nitrophenol is more acidic than ortho-methoxyphenol.
(iii) Cumene is a better starting material for the preparation of phenol.

14. The rate constant for a first order reaction is 60 s-1. How much time will it 3
take to reduce 1g of the reactant to 0.0625 g?

15. (i) Solutions of two electrolytes ‘A’ and ‘B’ are diluted. The limiting 3
molar conductivity of ‘B’ increases 1.5 times while that of ‘A’
increases 25times. Which of the two is a strong electrolyte? Justify
your answer.
(ii) The products of electrolysis of aqueous NaCl at the respective
electrodes are :
Cathode : H2
Anode : Cl2 and not O2. Explain.

16. (i) Write the expression for Freundlich’s equation to describe the 3
behaviour of adsorption from solution.
(ii) What causes charge on sol particles?
(iii) Name the promoter used in the Haber’s process for the manufacture
of ammonia.

17. An organic aromatic compound ‘A’ with the molecular formula C6H7N is 3
sparingly soluble in water. ‘A’ on treatment with dil HCI gives a water soluble
compound ‘B’. ‘A’ also reacts with chloroform in presence of alcoholic KOH
to form an obnoxious smelling compound ‘C’. ‘A’ reacts with benzene
sulphonyl chloride to form an alkali soluble compound ‘D’.’A’ reacts with
NaNO2 and HCl to form a compound ‘E’ which on reaction with phenol forms
an orange red dye ‘F’. Elucidate the structures of the organic compounds
from ‘A’ to ‘F’.

18. (i) Which vitamin deficiency causes rickets? 3


(ii) Name the base that is found in nucleotide of RNA only.
(iii) Glucose on reaction with acetic acid gives glucose penta acetate. What
does it suggest about the structure of glucose?

19. Name the type of reaction involved in the formation of the following 3
polymers from their respective monomers
(i) PVC.
(ii) Nylon6.
(iii) PHBV.

20. Describe the role of 3


(i) NaCN in the extraction of gold from its ore.
(ii) Cryolite in the extraction of aluminium from pure alumina.
(iii) CO in the purification of Nickel

21. A metal ion Mn+ having d4 valence electronic configuration combines with 3
three bidentate ligands to form a complex compound. Assuming  o  P :
(i) Write the electronic configuration of d4 ion.
(ii) What type of hybridisation will Mn+ ion has?
(iii) Name the type of isomerism exhibited by this complex.

22. The magnetic moments of few transition metal ions are given below: 3
Metal ion Magnetic moment(BM)
Sc3+ 0.00
2+
Cr 4.90
2+
Ni 2.84
Ti3+ 1.73
(at no. Sc = 21, Ti =22, Cr = 24, Ni = 28)
Which of the given metal ions :
(i) has the maximum number of unpaired electrons?
(ii) forms colourless aqueous solution?
(iii) exhibits the most stable +3 oxidation state?

OR

Consider the standard electrode potential values (M2+ / M) of the elements of


the first transition series.
Ti V Cr Mn Fe Co Ni Cu Zn
-1.63 -1.18 -0.90 -1.18 -0.44 -0.28 -0.25 +0.34 -0.76
Explain:
(i) Eo value for copper is positive.
(ii) Eo value of Mn is more negative as expected from the trend.
(iii) Cr2+ is a stronger reducing agent than Fe2+.

23. Ashwin observed that his friend Shubham was staying aloof, not playing with 4
friends and becoming easily irritable for some weeks. Ashwin told his teacher
about this, who, in turn, called Shubham’s parents and advised them to
consult a doctor. Doctor after examining Shubham prescribed antidepressant
drugs for him.
After reading the above passage, answer the following questions:
i) Name two antidepressant drugs.
ii) Mention the values shown by Ashwin.
iii) How should Shubham’s family help him other than providing medicine?
iv) What is the scientific explanation for the feeling of depression?

24. (a) Arrange the following in the order of property indicated against each 5
set:
(i) F2, Cl2, Br2, I2 (increasing bond dissociation enthalpy)
(ii) H2O, H2S, H2Se, H2Te (increasing acidic character)
(b) A colourless gas ‘A’ with a pungent odour is highly soluble in water
and its aqueous solution is weakly basic. As a weak base it
precipitates the hydroxides of many metals from their salt solution.
Gas ‘A’ finds application in detection of metal ions. It gives a deep
blue colouration with copper ions. Identify the gas ‘A’ and write the
chemical equations involved in the following:
(i) Gas ‘A’ with copper ions
(ii) Solution of gas ‘A’ with ZnSO4 solution.

OR

Answer the following questions


(a) Write the formula of the neutral molecule which is isoelectronic with
ClO-.
(b) Draw the shape of H2S2O7.
(c) Nitric acid forms an oxide of nitrogen on reaction with P4.Write the
formula of the stable molecule formed when this oxide undergoes
dimerisation.
(d) Bleaching action of chlorine is permanent. Justify.
(e) Write the disproportionation reaction of that oxoacid of nitrogen in
which nitrogen is in +3 oxidation state.

25. Write the products of the following reactions: 5

(b) Give simple chemical tests to distinguish between the following pairs of
compounds:
(i) Benzaldehyde and Benzoic acid
(ii) Propanal and Propanone
OR
(a) Account for the following:
(i) CH3CHO is more reactive than CH3COCH3 towards reaction with HCN.
(ii) 2-Fluorobutanoic acid is a stronger acid than 3-Fluorobutanoic acid.
(b) Write the chemical equations to illustrate the following name reactions:
(i) Etard reaction.
(ii) Rosenmund’s reaction.
(c) Give the mechanism of cyanohydrin formation when carbonyl
compounds react with HCN in the presence of alkali.
26. (i) Following is the schematic alignment of magnetic moments: 5

Identify the type of magnetism. What happens when these substances


are heated?
(ii) If the radius of the octahedral void is ‘r’ and radius of the atoms in
close packing is ‘R’. What is the relation between ‘r’ and ‘R’?
(iii) Tungsten crystallizes in body centred cubic unit cell. If the edge of the
unit cell is 316.5 pm. What is the radius of tungsten atom?
OR
(i) Identify the type of defect shown in the following figure:

What type of substances show this defect?


(ii) A metal crystallizes in a body centred cubic structure. If ‘a’ is the
edge length of its unit cell, ‘r’ is the radius of the sphere. What is
the relationship between ‘r’ and ‘a’?
(iii) An element with molar mass 63 g / mol forms a cubic unit cell
with edge length of 360.8 pm. If its density is 8.92 g/ cm3. What is
the nature of the cubic unit cell?
Class XII
Chemistry (Code – 043)
Sample Question Paper 2018-19

Time allowed: 3 Hours Max. Marks: 70

General Instructions:
(a) All questions are compulsory.
(b) Section A: Q.no. 1 to 5 are very short answer questions and carry 1 mark each.
(c) Section B: Q.no. 6 to 12 are short answer questions and carry 2 marks each.
(d) Section C: Q.no. 13 to 24 are also short answer questions and carry 3 marks each.
(e) Section D: Q.no. 25 to 27 are long answer questions and carry 5 marks each.
(f) There is no overall choice. However an internal choice has been provided in two questions of
one mark, two questions of two marks, four questions of three marks and all the three questions
of five marks weightage. You have to attempt only one of the choices in such questions.
(g) Use of log tables if necessary, use of calculators is not allowed.

Section-A

1. ZnO crystal on heating acquires the formula Zn 1+x O. Give reason. 1

OR
There is an increase in conductivity when Silicon is doped with Phosphorous. Give reason

2. Based on the type of dispersed phase, what type of colloids are micelles? 1

3. On the basis of crystal field theory, write the electronic configuration of d 6 in terms of t 2 g 1
and eg in an octahedral field when  o  P.

OR
Low spin configuration are rarely observed in tetrahedral coordination entity formation.
Explain

4. Identify the compound that on hydrogenation produces an optically active compound from 1
the following compounds:

H
Br Br
H
H2C CH3 CH3
H3C
(A) (B)

5. Write the name of the biodegradable polymer used in orthopaedic devices. 1

1
Section-B

6. Calculate the freezing point of a solution containing 8.1 g of HBr in 100 g of water, 2
assuming the acid to be 90 % ionized.
[Given: Molar mass Br = 80 g/mol, Kf water = 1.86 K kg / mol]

OR
Calculate the molality of ethanol solution in which the mole fraction of water is 0.88.

7. Identify the reaction and write the IUPAC name of the product formed: 2
(a)

(i) Br2 / Red phosphorous


CH3-CH2-COOH

(b)
O Cl

H2

Pd- BaSO4

OR
Write the structures and IUPAC names of the cross aldol condensation products only of
ethanal and propanal.

8. (a) Justify the role of tert-butyl peroxide in the polymerization of ethene. 2


(b) Write the structures of the monomers of the following polymer:
C6H5

CH2 - CH = CH - CH2 - CH2 -CH


n
9. Write the mechanism of acid dehydration of ethanol to yield ethane 2

10. For a certain chemical reaction variation in concentration [A] vs. time (s) plot is given
below:

(i) Predict the order of the given reaction?


(ii) What does the slope of the line and
intercept indicate?
(iii) What is the unit of rate constant k?

2
11. Draw the molecular structures of the following: 2
(a) Noble gas species which is isostructural with BrO3
(b) Dibasic oxoacid of phosphorus

12. (i) On the basis of the standard electrode potential values stated for acid solutions, predict 2
whether Ti4+ species may be used to oxidise Fe(II) to Fe(III)
Ti 4  e  Ti3 E o  0.01V
Fe 3  e  Fe 2 E o  0.77V

(ii) Based on the data arrange Fe2+, Mn2+ and Cr2+ in the increasing order of stability of +2
oxidation state.(Give a brief reason)
E o Cr3 / Cr 2   0.4V
E o Mn3 / Mn2   1.5V
E o Fe 3 / Fe 2   0.8V
Section-C

13. Niobium crystallises in body-centred cubic structure. If the atomic radius is 143.1 pm, 3
calculate the density of Niobium. (Atomic mass = 93u).

14. Give reasons for the following: 3


a. When 2g of benzoic acid is dissolved in 25 g of benzene, the experimentally
determined molar mass is always greater than the true value.
b. Mixture of ethanol and acetone shows positive deviation from Raoult’s Law.
c. The preservation of fruits by adding concentrated sugar solution protects against
bacterial action.

15. An alcohol A (C4H10O) on oxidation with acidified potassium dichromate gives acid B 3
(C4H8O2). Compound A when dehydrated with conc. H2SO4 at 443 K gives compound C.
Treatment of C with aqueous H2SO4 gives compound D (C4H10O) which is an isomer of
A. Compound D is resistant to oxidation but compound A can be easily oxidised. Identify
A, B, C and D. Name the type of isomerism exhibited by A and D

16. Which one of the following compounds will undergo faster hydrolysis reaction by S N1 3
mechanism? Justify your answer.
CH2Cl

or CH3CH2CH2Cl

OR
A compound is formed by the substitution of two chlorine atoms for two hydrogen atoms in
propane. Write the structures of the isomers possible. Give the IUPAC name of the isomer
which can exhibit enantiomerism.

3
17. Complete the following reactions : 3

(a)
+ H2N OH

(b)
KMnO4 , H2SO4


(c)
COOH Strong heating
+ NH3

COOH

18. Give reasons for the following: 3

(i) Use of aspartame as an artificial sweetener is limited to cold foods.


(ii) Metal hydroxides are better alternatives than sodium hydrogen carbonate
for treatment of acidity.
(iii) Aspirin is used in prevention of heart attacks.

19. (a) Name the branched chain component of starch. 3


(b) Ribose in RNA and deoxyribose in DNA differ in the structure around which carbon
atom?
(c) How many peptide linkages are present in a tripeptide?

OR
Give three reactions of glucose which cannot be explained by its chain structure

20. The following data were obtained during the first order thermal decomposition of N2O5(g) 3
at a constant volume:

2 N 2O5 ( g )  2 N 2O4 ( g )  O2 ( g )

S.No. Time (sec.) Total pressure(atm)


1. 0 0.5
2. 100 0.512

Calculate the rate constant

OR

Two reactions of the same order have equal pre exponential factors but their activation
energies differ by 24.9 kJ mol-1. Calculate the ratio between the rate constants of these
reactions at 27oC. (Gas constant R = 8.314 J K−1 mol−1)

4
21. (a) A colloidal sol is prepared by the given method in figure. What is the charge of AgI 3
colloidal particles in the test tube? How is the sol formed, represented?

(b) Explain how the phenomenon of adsorption finds application in Heterogeneous


catalysis.
(c) Which of the following electrolytes is the most effective for the coagulation of
Fe(OH)3 sol which is a positively charged sol ?
NaCl, Na2SO4, Na3PO4

22. Describe how the following steps can be carried out? 3

(a) Recovery of Gold from leached gold metal complex..


(b) Conversion of Zirconium iodide to pure Zirconium.
(c) Formation of slag in the extraction of copper.
(Write the chemical equations also for the reactions involved)

OR
Explain the use of the following:

a) NaCN in Froth Floatation Method.


b) Carbon monoxide in Mond process.
c) Coke in the extraction of Zinc from Zinc Oxide

23. Explain the following: 3

(a) Out of Sc3+, Co2+ and Cr3+ions, only Sc3+ is colourless in aqueous solutions.
(Atomic no.: Co = 27; Sc = 21 and Cr = 24)
(b) The E o Cu2 / Cu for copper metal is positive (+0.34), unlike the remaining
members of the first transition series
(c) La(OH)3 is more basic than Lu(OH)3.

24. A metal complex having composition Cr (NH3)4Cl2Br has been isolated in two forms A and 3
B. The form A reacts with AgNO3 to give a white precipitate readily soluble in dilute
aqueous ammonia whereas B gives a pale yellow precipitate soluble in concentrated
ammonia.

(i) Write the formulae of isomers A and B.


(ii) State the hybridisation of chromium in each of them.
(iii) Calculate the magnetic moment (spin only value) of the isomer A

5
Section-D

25. (a) Identify A-D 5


CH2NO2
CH2Cl
A Sn / HCl NaOH(aq)/Br2
B C

CH2NC

(b) Distinguish between the following pair of compounds:


(i) Aniline and Benzylamine.
(ii) Methylamine and Dimethylamine.
(c) Complete the following:
CH 3CH 2CN LiAlH
4  A 0C   B
0
/ HNO2

OR
(a) Account for the following:

(i) Direct nitration of aniline yields significant amount of meta derivative.


(ii) Primary aromatic amines cannot be prepared by Gabriel phthalimide
synthesis.

(b) Carry out the following conversions:


(i) Ethanoic acid into methanamine.
(ii) Aniline to p-Bromoaniline.

(c) Arrange the following in increasing order of basic strength:


Aniline, p-nitroaniline and p-toludine.

26. (a) A cell is prepared by dipping a zinc rod in 1M zinc sulphate solution and a silver 5
electrode in 1M silver nitrate solution. The standard electrode potential given:
E0Zn2+ / Zn = - 0.76 V, E0Ag+ / Ag = + 0.80 V
What is the effect of increase in concentration of Zn2+ on the Ecell?

(b) Write the products of electrolysis of aqueous solution of NaCl with platinum electrodes.

(c) Calculate e.m.f. of the following cell at 298 K:


Ni(s) / Ni2+ (0.01 M) // Cu2+ (0.1M) / Cu (s)
[ Given E0 Ni2+/ Ni = - 0.25 V , E0 Cu2+/Cu = + 0.34 V )
Write the overall cell reaction.

OR

6
(a) Apply Kohlrausch law of independent migration of ions, write the expression
to determine the limiting molar conductivity of calcium chloride.

(b) Given are the conductivity and molar conductivity of NaCl solutions at 298K
at different concentrations:

Concentration Conductivity Molar conductivity


M Scm-1 S cm2 mol-1
0.100 106.74 x 10 -4 106.7
0.05 55.53 x 10 -4 111.1
0.02 23.15 x 10 -4 115.8

Compare the variation of conductivity and molar conductivity of NaCl


solutions on dilution. Give reason.

(c) 0.1 M KCl solution offered a resistance of 100 ohms in a conductivity cell at
298 K. If the cell constant of the cell is 1.29 cm-1, calculate the molar
conductivity of KCl solution.

27. (a) Account for the following observations: 5

(i) SF4 is easily hydrolysed whereas SF6 is not easily hydrolysed


(ii) Chlorine water is a powerful bleaching agent.
(iii) Bi(V) is a stronger oxidising agent than Sb(V)

(b) What happens when

(i) White phosphorus is heated with concentrated NaOH solution in an


inert atmosphere of CO2.
(ii) XeF6 undergoes partial hydrolysis.
(Give the chemical equations involved).

OR

(a) What inspired N.Bartlett for carrying out reaction between Xe and PtF6?
(b) Arrange the following in the order of property indicated against each set:

(i) F2, I2, Br2, Cl2 (increasing bond dissociation enthalpy)


(ii) NH3, AsH3, SbH3, BiH3, PH3 (decreasing base strength)

(c) Complete the following equations:


(i) Cl 2  NaOH (cold and dilute ) 
(ii) Fe 3  SO2  H 2O 

7
SAMPLE PAPER XII 2019-20
CHEMISTRY

Time : 3 hrs. M. Marks : 70

General Instructions
(a) All questions are compulsory.
(b) Section A: Q.no. 1 to 20 are very short answer questions (objective type) and
carry 1 mark each.
(c) Section B: Q.no. 21 to 27 are short answer questions and carry 2 marks each.
(d) Section C: Q.no. 28 to 34 are long answer questions and carry 3 marks each.
(e) Section D: Q.no. 35 to 37 are also long answer questions and carry 5 marks each.
(f) There is no overall choice. However an internal choice has been provided in two
questions of two marks, two questions of three marks and all the three questions of
five marks weightage. You have to attempt only one of the choices in such
questions.
(g) Use log tables if necessary, use of calculators is not allowed.

SECTION - A

Read the given passage and answer the questions 1 to 5that follow:

A Lead storage battery is the most important type of secondary cell having a lead anode and a
grid of lead packed with PbO 2 as cathode. A 38% solution of sulphuric acid is used as
electrolyte. (Density=1.294 g mL -1) The battery holds 3.5 L of the acid. During the discharge
of the battery, the density of H2 SO4 falls to 1.139 g mL-1 . (20% H2SO4 by mass)
(1) Write the reaction taking place at the cathode when the battery is in use.
(2) How much electricity in terms of Faraday is required to carry out the reduction of
one mole of PbO2?
(3) What is the molarity of sulphuric acid before discharge?
(4) Lead storage battery is considered a secondary cell. Why?
(5) Write the products of electrolysis when dilute sulphuric acid is electrolysed using
Platinum electrodes.

Questions 6 to 10 are one word answers:

(6) Name the substance used as depressant in the separation of two sulphide ores in
Froth floatation method.
(7) Name the unit formed by the attachment of a base to 1 position of sugar in a
nucleoside.
(8) Name the species formed when an aqueous solution of amino acid is dissolved in
water?
(9) What type of reaction occurs in the formation of Nylon 6,6 polymer?
(10) Which of the following compoundswould undergo cannizzaro reaction:

Benzaldehyde, Cyclohexanone, 2- Methylpentanal.

Questions 11 to 15 are multiple choice questions:

(11) The IUPAC name of the compound shown below is:

(a) 2-bromo-6-chlorocyclohex-1-ene
(b) 6-bromo-2-chlorocyclohexene
(c) 3-bromo-1-chlorocyclohexene
(d) 1-bromo-3-chlorocyclohexene

(12) When one mole of CoCl3.5NH3 was treated with excess of silver nitrate
solution, 2 mol of AgCl was precipitated. The formula of the compound is:
(a) [Co(NH3)5Cl2]Cl
(b) [Co(NH3)5Cl]Cl2
(c) [Co(NH3)4Cl2] (NH3)Cl
(d) [Co(NH3)3Cl3] (NH3)2

(13) The absorption maxima of several octahedral complex ions are as follows:

S.No Compound max nm


1 [Co(NH3) 6]3+ 475
2 [Co(CN) 6]3- 310
3 [Co(H2O)6]3+ 490

The crystal field splitting is maximum for :


(a) [Co(H2O)6]3+

(b) [Co(CN)6]3-

(c) [Co(NH3)6]3+
(d) All the complex ions have the same splitting,  o ,

(14) Predict the number of ions produced per formula unit in an aqueous solution
of [Co(en)3]Cl3
(a) 4
(b) 3
(c) 6
(d) 2

(15) The incorrect statement about LDP is:

(a) It is obtained through the free radical addition of ethene.


(b) It consists of linear molecules.
(c) It is obtained by the H-atom abstraction.
(d) Peroxide is used as an initiator.

Questions 16 to 20 :
(A) Both assertion and reason are correct statements, and reason is the correct
explanation of the assertion.
(B) Both assertion and reason are correct statements, but reason is not the correct
explanation of the assertion.

(C) Assertion is correct, but reason is wrong statement.

(D) Assertion is wrong, but reason is correct statement.

16. Assertion: The two strands in double strand helix structure of DNA are
complementary to each other

Reason: Disulphide bonds are formed between specific pairs of bases

17. Assertion: Glucose reacts with hydroxylamine to form an oxime and alsoadds
a molecule of hydrogen cyanide to give cyanohydrin.

Reason: The carbonyl group is present in the open chain structure of


glucose.
18. Assertion: The acidic strength of halogen acids varies in the order
HF>HCl>HBr>HI

Reason: The bond dissociation enthalpy of halogen acids decreases in the


order HF>HCl>HBr>HI
19. Assertion: C2H5OH is a weaker base than phenol but is a stronger
nucleophile than phenol. (1)
Reason: In phenol the lone pair of electrons on oxygen is withdrawn
towards the ring due to resonance.
20. Assertion: Aryl halides undergo nucleophilic substitution reactions with
ease.
Reason:The carbon halogen bond in aryl halides has partial double bonds
character.

SECTION : B

21. Calculate the number of lone pairs on central atom in the following molecule and
predict the geometry.
XeF4

22. The rate of a reaction depends upon the temperature and is quantitatively expressed as

 Ea
k  Ae RT

i) If a graph is plotted between log k and 1/T, write the expression for the slope of
the reaction?
ii) If at under different conditions Ea1 and Ea2 are the activation energy of two
reactions If Ea1 = 40 J / mol and Ea2 = 80 J / mol. Which of the two has a larger
value of the rate constant?
23. The experimentally determined molar mass for what type ofsubstances is always
lower than the true value when water is used as solvent. Explain. Give one example
of such a substance and one example of a substance which does not show a large
variation from the true value.

24. Write structure of the products formed:


(a)

(b)
25. Draw one of the geometrical isomers of the complex [ Pt (en) 2 Cl2 ]2 which is optically
inactive. Also write the name of this entity according to the IUPAC nomenclature.
OR
Discuss the bonding in the coordination entity [CO( NH 3 ) 6 ]3 on the basis of
valence bond theory. Also, comment on the geometry and spin of the given entity.
(Atomic no. of Co= 27)

26. What is meant by Vapour phase refining? Write any one example of the process
which illustrates this technique, giving the chemical equations involved.

OR
Write and explain the reactions involved in the extraction of gold.
27. Which one of the following compounds will undergo hydrolysis at a faster rate
by SN1 mechanism? Justify.

CH2Cl

or CH3 CH2CH2Cl

SECTION: C

28. Calculate the freezing point of a solution containing 0.5 g KCl (Molar mass =
74.5 g/mol) dissolved in 100 g water, assuming KCl to be 92% ionized.
Kfof water = 1.86 K kg / mol.

29. For the reaction A + B →products, the following initial rates were obtained at
various given initial concentrations

S.No. [A] mol / L [B] mol / L Initial rate M/s


1. 0.1 0.1 0.05
2. 0.2 0.1 0.10
3. 0.1 0.2 0.05

Determine the half-life period.


OR
A first order reaction is 50 % complete in 50 minutes at 300 K and the same
reaction is again 50 % complete in 25 minutes at 350 K. Calculate activation
energy of the reaction.

30. Answer the following questions:

(a) Which of the following electrolytes is most effective for the coagulation of
AgI/Ag+ sol?
a. MgCl2, K2SO4, K4[Fe(CN)6]

(b) What happens when a freshly precipitated Fe(OH) 3 is shaken with a little
amount of dilute solution of FeCl3.
(c) Out of sulphur sol and proteins, which one forms macromolecular colloids?
31. Account for the following:

a) Moist SO2 decolourises KMnO4 solution.


b) In general interhalogen compounds are more reactive than halogens
(except fluorine).

c) Ozone acts as a powerful oxidizing agent

32. Identify the product formed when propan-1-ol is treated with Conc. H2 SO4 at
413 K . Write the mechanism involved for the above reaction.
33. (a) Give chemical tests to distinguish between the following pairs of compounds:

(i) Ethanal and Propanone.


(ii) Pentan-2-one and Pentan-3-one.

(b) Arrange the following compounds in increasing order of their acid strength:
Benzoic acid, 4- Nitrobenzoic acid, 3,4-Dinitrobenzoic acid,
4- Methoxybenzoic acid.
OR

Compare the reactivity of benzaldehyde and ethanal towards nucleophilic addition


reactions. Write the cross aldol condensation product between benzaldehyde and
ethanal.
34. Define and write an example for the following :
(a) Broad spectrum antibiotics.
(b) Analgesics

SECTION: D

35. (a) The e.m.f. of the following cell at 298 K is 0.1745 V


Fe (s) / Fe 2+ (0.1 M) // H + (x M)/ H2 (g) (1 bar)/ Pt (s)

E0   0.44V
Given : Fe2 / Fe
Calculate the H + ions concentration of the solution at the electrode where
hydrogen is being produced.
(b) Aqueous solution of copper sulphate and silver nitrate are electrolysed by 1
ampere current for 10 minutes in separate electrolytic cells. Will the mass of
copper and silver deposited on the cathode be same or different? Explain your
answer.

OR
(a) Calculate the degree of dissociation of 0.0024 M acetic acid if conductivity of this
solution is 8.0 × 10-5 S cm-1.

Given oH   349.6 S cm2 mol 1 ; oCH COO   40.9 S cm 2 mol 1


3

(b) Solutions of two electrolytes ‘A’ and ‘B’ are diluted. The limiting molar
conductivity of ‘B’ increases to a smaller extent while that of ‘A’ increases to a
much larger extent comparatively. Which of the two is a strong electrolyte?
Justify your answer.

36. An organic compound A’ with molecular formula C 7H7NO reacts with Br2/aqKOH to
give compound B’, which upon reaction with NaNO 2 & HCl at O°C gives C’.
Compound C’ on heating with CH3CH2OH gives a hydrocarbon D’. Compound B’ on
further reaction with Br2 water gives white precipitate of compound E’. Identify the
compound A, B, C, D&E; also justify your answer by giving relevant chemical
equations.

OR

(a) How will you convert:


(i) Aniline into Fluorobenzene.
(ii) Benzamide into Benzylamine.

(iii) Ethanamine to N,N-Diethylethanamine.

(b) Write the structures of A and B in the following:


i)

OH- NaOH + Br2


CH3CH2CN A B
Partial hydrolysis
ii)

i) KCN HNO2
CH3CH2Br A B
ii) LiAlH4 O0C

37. (a) When a chromite ore (A) is fused with an aqueous solution of
sodium carbonate in free excess of air, a yellow solution of compound
(B) is obtained. This solution is filtered and acidified with sulphuric
acid to form compound (C). Compound (C) on treatment with solution
of KCl gives orange crystals of compound (D). Write the chemical
formulae of compounds A to D.
(b) Describe the cause of the following variations with respect to lanthanoids
and actinoids:
(i) Greater range of oxidation states of actinoids as compared to
lanthanoids.
(ii) Greater actinoid contraction as compared to lanthanoid
contraction.
(iii) Lower ionisation enthalpy of early actinoids as compared to the
early lanthanoids.
OR
(a) What happens when
(i) Manganate ions ( MnO42 ) undergoes disproportionation reaction in
acidic medium?
(ii) Lanthanum is heated with Sulphur?

(b) Explain the following trends in the properties of the members of the First
series of transition elements:
(i) E o ( M 2 / M ) value for copper is positive(+0.34 V) in contrast to the
other members of the series.
(ii) Cr2+ is reducing while Mn3+ is oxidising, though both have d4
configuration.
(iii) The oxidising power in the series increases in the
order VO2  Cr2O72  MnO4 .
SAMPLE PAPER 1
CHEMISTRY THEORY (043)
MM:70 Time: 3 Hours

General Instructions:
Read the following instructions carefully.
a) There are 33 questions in this question paper. All questions are compulsory.
b) Section A: Q. No. 1 to 16 are objective type questions. Q. No. 1 and 2 are passage based
questions carrying 4 marks each while Q. No. 3 to 16 carry 1 mark each.
c) Section B: Q. No. 17 to 25 are short answer questions and carry 2 marks each.
d) Section C: Q. No. 26 to 30 are short answer questions and carry 3 marks each.
e) Section D: Q. No. 31 to 33 are long answer questions carrying 5 marks each.
f) There is no overall choice. However, internal choices have been provided.
g) Use of calculators and log tables is not permitted.

SECTION A (OBJECTIVE TYPE)


1. Read the passage given below and answer the following questions: (1x4=4)

An efficient, aerobic catalytic system for the transformation of alcohols into carbonyl compounds
under mild conditions, copper-based catalyst has been discovered. This copper-based catalytic
system utilizes oxygen or air as the ultimate, stoichiometric oxidant, producing water as the only
by-product

A wide range of primary, secondary, allylic, and benzylic alcohols can be smoothly oxidized to
the corresponding aldehydes or ketones in good to excellent yields. Air can be conveniently used
instead of oxygen without affecting the efficiency of the process. However, the use of air requires
slightly longer reaction times.
This process is not only economically viable and applicable to large-scale reactions, but it is also
environmentally friendly.
(Reference:Ohkuma, T., Ooka, H., Ikariya, T., & Noyori, R. (1995). Preferential hydrogenation of
aldehydes and ketones. Journal of the American Chemical Society, 117(41), 10417-10418.)
The following questions are multiple choice questions. Choose the most appropriate answer:
(i)The Copper based catalyst mention in the study above can be used to convert:
a) propanol to propanonic acid
b) propanone to propanoic acid
c) propanone to propan-2-ol
d) propan-2-ol to propanone

(ii)The carbonyl compound formed when ethanol gets oxidised using this copper-based catalyst
can also be obtained by ozonolysis of:
a) But-1-ene
b) But-2-ene
c) Ethene
d) Pent-1-ene
OR

Which of the following is a secondary allylic alcohol?


a) But-3-en-2-ol
b) But-2-en-2-ol
c) Prop-2-enol
d) Butan-2-ol
(iii) Benzyl alcohol on treatment with this copper-based catalyst gives a compound ‘A’ which
on reaction with KOH gives compounds ‘B’ and ‘C’. Compound ‘B’ on oxidation with
KMnO4- KOH gives compound ‘C’. Compounds ‘A’, ‘B’ and ‘C’ respectively are :
a) Benzaldehyde, Benzyl alcohol, potassium salt of Benzoic acid
b) Benzaldehyde, potassium salt of Benzoic acid, Benzyl alcohol
c) Benzaldehyde, Benzoic acid, Benzyl alcohol
d) Benzoic acid, Benzyl alcohol, Benzaldehyde
(iv) An organic compound ‘X’ with molecular formula C3H8O on reaction with this
copper based catalyst gives compound ‘Y’ which reduces Tollen’s reagent. ‘X’ on
reaction with sodium metal gives ‘Z’ . What is the product of reaction of ‘Z’ with 2-
chloro-2-methylpropane?
a) CH3CH2 CH2OC(CH3)3
b) CH3CH2OC(CH3)3
c) CH2=C(CH3)2
d) CH3CH2 CH=C(CH3)2

Read the passage given below and answer the following questions: (1x4=4)

The amount of moisture that leather adsorbs or loses is determined by temperature, relative
humidity, degree of porosity, and the size of the pores. Moisture has great practical significance
because its amount affects the durability of leather, and in articles such as shoes, gloves and other
garments, the comfort of the wearer. High moisture content accelerates deterioration and promotes
mildew action. On the other hand, a minimum amount of moisture is required to keep leather
properly lubricated and thus prevent cracking.
The study indicates that adsorption of moisture by leather is a multi-molecular process and is
accompanied by low enthalpies of adsorption. Further at 75-percent relative humidity, the
adsorption is a function of surface area alone.
Hide is tanned to harden leather. This process of tanning occurs due to mutual coagulation of
positively charged hide with negatively charged tanning material. Untanned hide and chrome-
tanned leathers have the largest surface areas. The leathers tanned with vegetable tanning
materials have smaller surface areas since they are composed of less hide substance and the
capillaries are reduced to smaller diameters, in some cases probably completely filled by tanning
materials. The result of the study indicated that untanned hide and chrome-tanned leather adsorb
the most water vapour.
(Source:Kanagy, J. R. (1947). Adsorption of water vapor by untanned hide and various leathers at
100 F. Journal of Research of the National Bureau of Standards, 38(1), 119-128.)
2. In these questions (Q. No 5-8 , a statement of assertion followed by a statement of reason
is given. Choose the correct answer out of the following choices.
a) Assertion and reason both are correct statements and reason is correct explanation for
assertion.
b) Assertion and reason both are correct statements but reason is not correct explanation for
assertion.
c) Assertion is correct statement but reason is wrong statement.
d) Assertion is wrong statement but reason is correct statement.

(i) Assertion: Vegetable tanned leather cannot adsorb a large amount of moisture.
Reason: Porous materials have higher surface area.

(ii) Assertion: Animal hide soaked in tannin results in hardening of leather.


Reason: Tanning occurs due to mutual coagulation.

(iii) Assertion: Adsorption of moisture by leather is physisorption.


Reason: It is a multimolecular process and is accompanied by low enthalpies of
adsorption
(iv) Assertion: Leathers tanned with vegetable tanning materials have smaller surface areas
Reason: The capillaries present in leather are reduced to smaller diameters
OR
Assertion: Leather absorbs different amount of moisture.
Reason: Some moisture is necessary to prevent cracking of leather.

Following questions (No. 3 -11) are multiple choice questions carrying 1 mark each:
3 Which of the following option will be the limiting molar conductivity of CH3COOH if the
limiting molar conductivity of CH3COONa is 91 Scm2mol-1? Limiting molar conductivity
for individual ions are given in the following table.

S.No Ions limiting molar conductivity / Scm2mol-1


1 H+ 349.6

2 Na+ 50.1
3 K+ 73.5
4 OH- 199.1
a) 350 Scm2mol-1
b) 375.3 Scm2mol-1
c) 390.5 Scm2mol-1
d) 340.4 Scm2mol-1
4. Curdling of milk is an example of:
a) breaking of peptide linkage
b) hydrolysis of lactose
c) breaking of protein into amino acids
d) denauration of proetin
OR
Dissachrides that are reducing in nature are:
a) sucrose and lactose
b) sucrose and maltose
c) lactose and maltose
d) sucrose, lactose and maltose

5.When 1 mole of benzene is mixed with 1 mole of toluene The vapour will contain: (Given :
vapour of benzene = 12.8kPa and vapour pressure of toluene = 3.85 kPa).
a) equal amount of benzene and toluene as it forms an ideal solution
b) unequal amount of benzene and toluene as it forms a non ideal solution
c) higher percentage of benzene
d) higher percentage of toluene

6.Which of the following is the reason for Zinc not exhibiting variable oxidation state
a) inert pair effect
b) completely filled 3d subshell
c) completely filled 4s subshell
d) common ion effect
OR
Which of the following is a diamagnetic ion: (Atomic numbers of Sc, V, Mn and Cu are 21, 23,
25 and 29 respectively)
a) V2+
b) Sc3+
c) Cu2+
d) Mn3+

7. Propanamide on reaction with bromine in aqueous NaOH gives:


a) Propanamine
b) Ethanamine
c) N-Methyl ethanamine
d) Propanenitrile
OR
IUPAC name of product formed by reaction of methyl amine with two moles of ethyl chloride
a) N,N-Dimethylethanamine
b) N,N-Diethylmethanamine
c) N-Methyl ethanamine
d) N-Ethyl - N-methylethanamine

8.Ambidentate ligands like NO2- and SCN- are :


a) unidentate
b) didentate
c) polydentate
d) has variable denticity
OR
The formula of the coordination compound Tetraammineaquachloridocobalt(III) chloride is
a) [Co(NH3)4(H2O)Cl]Cl2
b) [Co(NH3)4(H2O)Cl]Cl3
c) [Co(NH3)2(H2O)Cl]Cl2
d) [Co(NH3)4(H2O)Cl]Cl

9.Which set of ions exhibit specific colours? (Atomic number of Sc = 21, Ti = 22, V=23, Mn =
25, Fe = 26, Ni = 28 Cu = 29 and Zn =30)

a) Sc3+, Ti4+, Mn3+


b) Sc3+, Zn2+, Ni2+
c) V3+, V2+, Fe3+
d) Ti3+, Ti4+, Ni2+
10. Identify A,B,C and D:

a) A = C2H4, B= C2H5OH, C= C2H5NC, D= C2H5CN


b) A= C2H5OH, B= C2H4, C = C2H5CN, D=C2H5NC
c) A = C2H4, B= C2H5OH, C= C2H5CN, D= C2H5NC
d) A= C2H5OH, B= C2H4, C= C2H5NC, D= C2H5CN

11.The crystal showing Frenkel defect is :


a)

b)

c)

d)
In the following questions (Q. No. 12 - 16) a statement of assertion followed by a statement of
reason is given. Choose the correct answer out of the following choices.
a) Assertion and reason both are correct statements and reason is correct explanation for
assertion.
b) Assertion and reason both are correct statements but reason is not correct explanation for
assertion.
c) Assertion is correct statement but reason is wrong statement.
d) Assertion is wrong statement but reason is correct statement.

12.Assertion: The two strands of DNA are complementary to each other


Reason: The hydrogen bonds are formed between specific pairs of bases.

13.Assertion: Ozone is thermodynamically stable with respect to oxygen.


Reason:Decomposition of ozone into oxygen results in the liberation of heat

14.Assertion: Aquatic species are more comfortable in cold waters rather than in warm waters.
Reason:Different gases have different KH values at the same temperature
OR
Assertion: Nitric acid and water form maximum boiling azeotrope.
Reason: Azeotropes are binary mixtures having the same composition in liquid and vapour
phase.
15. Assertion: Carboxylic acids are more acidic than phenols.
Reason: Phenols are ortho and para directing.

16.Assertion: Methoxy ethane reacts with HI to give ethanol and iodomethane


Reason: Reaction of ether with HI follows SN2 mechanism

SECTION B

The following questions, Q.No 17 – 25 are short answer type and carry 2 marks each.

17. With the help of resonating structures explain the effect of presence of nitro group at ortho
position in chlorobenzene.
OR
Carry out the following conversions in not more than 2 steps:
(i)Aniline to chlorobenzene
(ii)2-bromopropane to 1- bromopropane

18. A glucose solution which boils at 101.04 oC at 1 atm. What will be relative lowering of
vapour pressure of an aqueous solution of urea which is equimolal to given glucose
solution? (Given: Kb for water is 0.52 K kg mol-1)

19. (i) Using crystal field theory, write the electronic configuration of iron ion in the following
complex ion. Also predict its magnetic behaviour :
[Fe(H2O)6]2+
(ii)Write the IUPAC name of the coordination complex: [CoCl2(en)2]NO3
OR
2-
(i)Predict the geometry of [Ni(CN)4]
(ii)Calculate the spin only magnetic moment of [Cu(NH3)4]2+ ion.

20. For a reaction the rate law expression is represented as follows:


Rate = k [A][B]1/2
i. Interpret whether the reaction is elementary or complex. Give reason to support your
answer.
ii. Write the units of rate constant for this reaction if concentration of A and B is
expressed in moles/L.
OR
The following results have been obtained during the kinetic studies of the reaction:
P + 2Q  R + 2S

Init. Rate of Formation


Exp. Initial P(mol/L) Initial Q (mol/L)
of R (M min-1)

1 0.10 0.10 3.0 x 10 -4

2 0.30 0.30 9.0 x 10 -4

3 0.10 0.30 3.0 x 10 -4

4 0.20 0.40 6.0 x 10 -4

Determine the rate law expression for the reaction.


21. The C-14 content of an ancient piece of wood was found to have three tenths of that in
living trees. How old is that piece of wood? (log 3= 0.4771, log 7 = 0.8540 , Half-life of
C-14 = 5730 years )

22. When 3-methylbutan-2-ol is treated with HBr, the following reaction takes place:

Give a mechanism for this reaction.

23. Give the formula and describe the structure of a noble gas species which
is isostructural with IF6-.

24. The following haloalkanes are hydrolysed in presence of aq KOH.


(i) 2- Chlorobutane (ii) 2-chloro-2-methylpropane
Which of the above is most likely to give a racemic mixture? Justify your answer.
25.Atoms of element P form ccp lattice and those of the element Q occupy 1/3rd of tetrahedral
voids and all octahedral voids. What is the formula of the compound formed by the elements
P and Q?

SECTION C

Q.No 26 -30 are Short Answer Type II carrying 3 mark each.


26. Give reasons for the following:
i. Transition elements act as catalysts
ii. It is difficult to obtain oxidation state greater than two for Copper.
iii. Cr2O72- is a strong oxidising agent in acidic medium whereas WO3 and MoO3 are not.

OR
Observed and calculated values for the standard electrode potentials of elements from Ti to Zn in
the first reactivity series are depicted in figure (1):
FIGURE 1 (source NCERT)
Explain the following observations:
i. The general trend towards less negative Eo values across the series
ii. The unique behaviour of Copper
iii. More negative Eo values of Mn and Zn

27. Arrange the following in increasing order of property specified:

i. Aniline, ethanamine, 2-ethylethanamine (solubility in water)


ii. Ethanoic acid, ethanamine, ethanol (boiling point)
iii. Methanamine, N, N- dimethylmethanamine and N- methylmethanamine (basic strength in
aqueous phase)
OR
i. Give a chemical test to distinguish between N-methylethanamine and N,N-dimethyl
ethanamine.
ii. Write the reaction for catalytic reduction of nitrobenzene followed by reaction of
product so formed with bromine water.
iii. Out of butan-1-ol and butan-1-amine, which will be more soluble in water and why?

28. A metal crystallizes into two cubic system-face centred cubic (fcc) and body centred cubic
(bcc) whose unit cell lengths are 3.5 and 3.0Å respectively. Calculate the ratio of densities of
fcc and bcc.
29. Three amino acids are given below:
Alanine CH3CH(COOH)(NH2) Aspartic acid HOOC-CH2CH(COOH)(NH2) and Lysine H2N-
(CH2)4-CH(COOH)(NH2)
i. Make two tripeptides using these amino acids and mark the peptide linkage in both cases.
ii. Represent Alanine in the zwitter ionic form.
30. i. Arrange the following in decreasing order of bond dissociation enthalpy
F2 , Cl2 , Br2 , I2
ii. Bi does not form pπ-pπ bonds. Give reason for the observation.
iii.Electron gain enthalpy of oxygen is less negative than sulphur. Justify

SECTION D

Q.No 31 to 33 are long answer type carrying 5 marks each.

31. (i) Answer the following questions: (2+3)


a) Write the balanced chemical reaction for reaction of Cu with dilute HNO3.
b) Draw the shape of ClF3
(ii)‘X’ has a boiling point of 4.2K, lowest for any known substance. It is used as a diluent for
oxygen in modern diving apparatus. Identify the gas ‘X’. Which property of this gas makes it
usable as diluent? Why is the boiling point of the gas ‘X’ so low?
OR
(i) Answer the following questions: (2+3)
a) Arrange the following in the increasing order of thermal stability:
H2O, H2S, H2Se, H2Te
b)Give the formula of the brown ring formed at the interface during the ring test for nitrate.
(ii) A greenish yellow gas ‘A’ with pungent and suffocating odour, is a powerful bleaching
agent. ‘A’ on treatment with dry slaked lime it gives bleaching powder. Identify ‘A’ and
explain the reason for its bleaching action. Write the balanced chemical equation for the
reaction of ‘A’ with hot and concentrated NaOH.
32. An organic compound ‘A’ C8H6 on treatment with dilute H2SO4 containing mercuric sulphate
gives compound ‘B’. This compound ‘B’ can also be obtained from a reaction of benzene with
acetyl chloride in presence of anhy AlCl3. ‘B’ on treatment with I2 in aq. KOH gives ‘C’ and a
yellow compound ‘D’. Identify A, B, C and D. Give the chemical reactions involved. (5)

OR
(i) Write the reaction for cross aldol condensation of acetone and ethanal.
(ii) How will you carry out the following conversions:
a) Benzyl alcohol to phenyl ethanoic acid
b) Propanone to propene
c) Benzene to m-Nitroacetophenone
33. (i) State Kohlrausch law. (1+4)
(ii) Calculate the emf of the following cell at 298 K:
Al(s)/Al3+ (0.15M)//Cu2+(0.025M) /Cu(s)
(Given Eo(Al3+/Al) = -1.66 V, Eo(Cu 2+/Cu) = 0.34V, log 0.15 = -0.8239, log 0.025 = -
1.6020)
OR
(i) On the basis of Eo values identify which amongst the following is the strongest
oxidising agent (1+4)
Cl2(g) + 2 e– 2Cl- Eo= +1.36 V,
MnO4– + 8H+ + 5e– → Mn2+ + 4H2O Eo= +1.51 V
Cr2O72– + 14H+ + 6e– → 2Cr3+ + 7H2O Eo= +1.33 V

(ii) The following figure 2, represents variation of (Λm ) vs √c for an electrolyte. Here Λm
is the molar conductivity and c is the concentration of the electrolyte.

Figure 2
a) Define molar conductivity
b) Identify the nature of electrolyte on the basis of the above plot. Justify your answer.
c) Determine the value of Λmo for the electrolyte.
d) Show how to calculate the value of A for the electrolyte using the above graph.
Sample Question Paper 2021-22
Term 1
Subject: Chemistry (043)

Time: 90 Minutes Max. Marks: 35

General Instructions:

1. The Question Paper contains three sections.

2. Section A has 25 questions. Attempt any 20 questions.

3. Section B has 24 questions. Attempt any 20 questions.

4. Section C has 6 questions. Attempt any 5 questions.

5. All questions carry equal marks.

6. There is no negative marking.

SECTION A
This section consists of 25multiple choice questions with overall choice to attempt any 20
questions. In case more than desirable number of questions are attempted, ONLY first 20 will
be considered for evaluation.

1.Which of the following statements is true:


(a)Melting point of Phosphorous is less than that of Nitrogen
(b)N2 is highly reactive while P4 is inert
(c)Nitrogen shows higher tendency of catenation than P
(d)N-N is weaker than P-P

2. Which of the following is a non-stoichiometric defect?


(a)Frenkel defect
(b)Schottky defect
(c)metal deficiency defect
(d)interstitial defect

3. Identify the law which is stated as:


“For any solution, the partial vapour pressure of each volatile component in the solution is
directly proportional to its mole fraction.”
(a)Henry’s law
(b) Raoult’s law
(c)Dalton’s law
(d)Gay-Lussac's Law
4. Pink colour of LiCl crystals is due to:
(a) Schottky defect
(b)Frenkel defect
(c) Metal excess defect
(d) Metal deficiency defect

5. Which of the following isomer has the highest meltingpoint:


(a) 1,2-dicholorbenzene
(b) 1,3 -dichlorobenzene
(c) 1,4-dicholorbenzene
(d) all isomers have same melting points

6. Which one of the following reactions is not explained by the open chain
Structureof glucose:
(a) Formation of pentaacetate of glucose with acetic anhydride.
(b) formation of addition product with 2,4 DNP reagent
(c) Silver mirror formation with Tollen’s reagent
(d) existence of alpha and beta forms of glucose.

7. Williamson’s synthesis of preparing dimethyl ether is an:


(a) SN1 reaction
(b) Elimination reaction
(c) SN2 reaction
(d) Nucleophilic addition reaction

8. Chlorine water loses its yellow colour on standing because:


(a) HCl gas is produced, due to the action of sunlight.
(b) a mixture of HOCl and HCl is produced in the presence of light
(c) HOCl and hydrogen gas is produced
(d) a mixture of HCl and ClO3 is produced, due to the action of sunlight

9. During dehydration of alcohols to alkenes by heating with concentrated H2SO4, the


initiation step is:
(a) protonation of alcohol molecule
(b) formation of carbocation
(c) elimination of water
(d) formation of an ester

10. Amorphous solids are:


(a) isotropic
(b)anisotropic
(c) isotopic
(d) isomeric

11. Which of the following reactions is used to prepare salicylaldehyde?


(a) Kolbe’s reaction
(b) Etard reaction
(c) Reimer- Tiemann reaction
(d) Stephen’s reduction.
12. Which of the following is an example of a solid solution?
(a)sea water
(b)sugar solution
(c)smoke
(d)22 carat gold

13. The boiling points of alcohols are higher than those of hydrocarbons of comparable
masses due to:
(a) Hydrogen bonding
( b) Ion – dipole interaction
(c) Dipole- dipole interaction
(d) Van der Waal’s forces.

14. Which of the following has the lowest boiling point:


(a)H2O
(b)H2S
(c)H2Se
(d)H2Te

15. Which of the following statement is correct:


(a)Fibrous proteins are generally soluble in water
(b)Albumin is an example of fibrous proteins
(c)In fibrous proteins, the structure is stabilised by hydrogen bonds and disulphide bonds
(d)pH does not affect the primary structure of protein.

16. Major product obtained on reaction of 3-Phenyl propene with HBr in presence of organic
peroxide
(a)3- Phenyl 1- bromopropane
(b) 1 –Phenyl -3- bromopropane
(c) 1-Phenyl -2-bromopropane
(d) 3-Phenyl -2- bromopropane

17. Which of the following is a correct statement for C2H5Br?


(a) It reacts with metallic Na to give ethane.
(b) It gives nitroethane on heating with aqueous solution of AgNO2
(c) It gives C2H5OH on boiling with alcoholic potash.
(d) It forms diethylthioether on heating with alcoholic KSH.

18.Covalency of nitrogen is restricted to:


(a)2
(b)3
(c)4
(d)5

19.Solubility of gases in liquids decreases with rise in temperature because dissolution is an:
(a)endothermic and reversible process
(b)exothermic and reversible process
(c)endothermic and irreversible process
(d) exothermic and irreversible process
20.All elements of Group 15 show allotropy except:
(a)Nitrogen
(b)Arsenic
(c)Antimony
(d)Bismuth

21.Which of the following is a polysaccharide?


(a)glucose
(b)maltose
(c)glycogen
(d)lactose

22. Substance having the lowest boiling point:


(a)Hydrogen
(b)Oxygen
(c)Nitrogen
(d) Helium

23.Lower molecular mass alcohols are:


(a)miscible in limited amount of water
(b) miscible in excess of water
(c) miscible in water in all proportions
(d) immiscible in water

24.Maximum oxidation state exhibited by Chlorine is:


(a) +1
(b) +3
(c)+5
(d)+7

25.In which of the following cases blood cells will shrink:


(a)when placed in water containing more than 0.9% (mass/ volume) NaCl solution.
(b)when placed in water containing less than 0.9% (mass /volume) NaCl solution.
(c)when placed in water containing 0.9% (mass/volume) NaCl solution.
(d)when placed in distilled water.

SECTION B
This section consists of 24multiple choice questions with overall choice to attempt any 20
questions. In case more than desirable number of questions are attempted, ONLY first 20 will
be considered for evaluation.

26. How much ethyl alcohol must be added to 1 litre of water so that the solution will freeze
at– 14°C ? (Kf for water = 1.86°C/mol)
(a) 7.5 mol
(b)8.5 mol
(c)9.5 mol
(d)10.5 mol
27. Which reagents are required for one step conversion of chlorobenzene to toluene?
(a) CH3Cl / AlCl3
(b) CH3Cl, Na, Dry ether
(c)CH3Cl/Fe dark
(d) NaNO2/ HCl /0-50C

28. On partial hydrolysis, XeF6 gives:


(a) XeO3 +4HF
(b) XeO2F + HF
(c) XeOF4+ H2
(d) XeO2F2 + 4HF

29. Which one of the following statement is correct about sucrose :


(a) It can reduce tollen’s reagent however cannot reduce fehling’s reagent
(b) It undergoes mutarotation like glucose and fructose
(c) It undergoes inversion in the configuration on hydrolysis
(d) It is laevorotatory in nature .

30. Phenol does not undergo nucleophilic substitution reaction easily due to:
(a) acidic nature of phenol
(b) partial double bond character of C-OH bond
(c) partial double bond character of C-C bond
(d)instability of phenoxide ion

31. Which of the following has highest ionisation enthalpy?


(a)Nitrogen
(b)Phosphorus
(c)Oxygen
(d)Sulphur
32. Metal M ions form accp structure. Oxide ions occupy ½ octahedral and ½ tetrahedral
voids. What is the formula of the oxide?
(a)MO
(b)MO2
(c)MO3
(d) M2O3

33. The reaction of toluene with Cl2 in presence of FeCl3 gives ‘X’ while theof toluene with
Cl2in presence of light gives ‘Y’. Thus ‘X’ and ‘Y’are:
(a) X = benzyl chloride Y = o and p – chlorotoluene
(b) X = m – chlorotoluene Y = p – chlorotoluene
(c) X = o and p–chlorotoluene Y = trichloromethylbenzene
(d) X= benzyl chloride, Y = m-chlorotoluene
34.Ozone is a/ an __________ molecule and the two O-O bond lengths
in ozone are (i)_______-and (ii) ____________
(a) linear ,110pm ; 148pm
(b) angular, 110pm ; 148pm
(c)linear, 128pm ; 128pm
(d)angular, 128pm ; 128pm

35. Water retention or puffiness due to high salt intake occurs due to:
(a)diffusion
(b)vapour pressure difference
(c) osmosis
(d)reverse osmosis

36. In the following reaction,identify A and B:

C6H12O6 Acetic anhydride A

Conc. nitric acid

(a) A= COOH-(CH2)4 -COOH, B= OHC-(CHOCOCH3)4 -CH2OCOCH3


(b) A= COOH-(CH2)4 -CHO , B= OHC-(CHOCOCH3)4 -CH2OCOCH3
(c) A= OHC-(CHOCOCH3)3-CH2OCOCH3 B= COOH-(CH2)4 -CHO ,
(d) A= OHC-(CHOCOCH3)4-CH2OCOCH3 B= COOH-(CH2)4 -COOH

37. In lake test for Al3+ ions, there is the formation of coloured ‘floating lake’. It is due to:
(a)Absorption of litmus by [Al(OH)4]-
(b)Absorption of litmus by Al(OH)3
(c)Adsorption of litmus by [Al(OH)4]-
(d) Adsorption of litmus by Al(OH)3

38. A unit cell of NaCl has 4 formula units. Its edge length is 0.50 nm. Calculate the density
if molar mass of NaCl = 58.5 g/mol.
(a) 1 g/cm3
(b)2 g/cm3
(c) 3 g/cm3
(d)4g/cm3

39.Which one of the following are correctly arranged on the basis of the property indicated:
(a) I2< Br2<F2<Cl2 [ increasing bond dissociation enthalpy]
(b) H2O > H2S<H2Te<H2Se [ increasing acidic strength]
(c) NH3 < N2O< NH2OH<N2O5 [ increasing oxidation state]
(d) BiH3<SbH3<AsH3<PH3<NH3 [ increasing bondangle]
40.. What would be the reactant and re
reagent used to obtain 2, 4-dimethyl pentan--3-ol?
(a) Propanal and propyl magnesium bromide
(b) 3-methylbutanal and 2-methyl
methyl magnesium iodide
(c) 2-dimethylpropanone
dimethylpropanone and methyl magnesium iodide
(d) 2- methylpropanal and isopropyl magnesium iodide

hydroxy benzyl alcohol when reacted with PCl3 gives the product as (IUPAC name)
41. o-hydroxy
(a) o- hydroxy benzyl chloride
(b) 2- chloromethylphenol
(c) o-chloromethylchlorobenzene
chloromethylchlorobenzene
(d) 4-hydroxymethylphenol

42.. Which of the following statements is true:


(a)Ammonia is the weakest reducing agent and the strongest base among Group 15 hydrides.
(b) Ammonia is the strongest reducing agent as well as the strongest base among Group 15
hydrides.
(c)Ammonia is the weakest reducing agent as well as the weakest base among Group 15
hydrides.
(d) Ammonia is the strongest reducing agent and the weakest base among Group 15 hydrides.

43.Identify the secondary alcohols


ols from the following set:
(i)CH3CH2CH(OH)CH3
(ii) (C2H5)3COH
(iii)

(iv)

(a)(i) and (iv)


(b)(i) and (iii)
(c)(i) and (ii)
(d)(i), (iii) and (iv)

44.Alkenes
Alkenes decolourise bromine water in presence of CCl4 due to formation of:
(a)allyl bromide
(b)vinyl bromide
(c)bromoform
(d)vicinal dibromide
45. Given below are two statements labelled as Assertion (A) and Reason (R)
Assertion (A): Electron gain enthalpy of oxygen is less than that of Flourine but greater than
Nitrogen.
Reason (R): Ionisation enthalpies of the elements follow the order Nitrogen > Oxygen >
Fluorine
Select the most appropriate answer from the options given below:
(a) Both A and R are true and R is the correct explanation of A
(b) Both A and R are true but R is not the correct explanation of A.
(c)A is true but R is false.
(d) A is false but R is true.

46. Given below are two statements labelled as Assertion (A) and Reason (R)
Assertion (A): Alkyl halides are insoluble in water.
Reason (R): Alkyl halides have halogen attached to sp3 hybrid carbon.
Select the most appropriate answer from the options given below:
(a) Both A and R are true and R is the correct explanation of A
(b) Both A and R are true but R is not the correct explanation of A.
(c)A is true but R is false.
(d) A is false but R is true.

47. Given below are two statements labelled as Assertion (A) and Reason (R)
Assertion(A): Molarity of a solution changes with temperature.
Reason (R): Molarity is a colligative property.
Select the most appropriate answer from the options given below:
(a) Both A and R are true and R is the correct explanation of A
(b) Both A and R are true but R is not the correct explanation of A.
(c)A is true but R is false.
(d) A is false but R is true.

48. Given below are two statements labelled as Assertion (A) and Reason (R)
Assertion(A):SO2 is reducing while TeO2 is an oxidising agent.
Reason(R):Reducing property of dioxide decreases from SO2 to TeO2.
Select the most appropriate answer from the options given below:
(a) Both A and R are true and R is the correct explanation of A
(b) Both A and R are true but R is not the correct explanation of A.
(c)A is true but R is false.
(d) A is false but R is true.

49.Given below are two statements labelled as Assertion (A) and Reason (R)
Assertion (A):Cryoscopic constant depends on nature of solvent.
Reason(R ):Cryoscopic constant is a universal constant.
Select the most appropriate answer from the options given below:
(a) Both A and R are true and R is the correct explanation of A
(b) Both A and R are true but R is not the correct explanation of A.
(c)A is true but R is false.
(d) A is false but R is true.
SECTION C
This section consists of 6multiple choice questions with an overall choice to attempt any5. In
case more than desirable number of questions are attempted, ONLY first 5 will be considered
for evaluation.

50.Match the following:


I II
(i)Amino acids (A)protein
(ii)Thymine (B)Nucleic acid
(iii)Insulin ( C)DNA
(iv)phosphodiester linkage (D)Zwitter ion
(v) Uracil

Which of the following is the best matched options?


(a) i-A, v- D, iii- C, iv-B
(b) i-D, ii-C, iii- A, iv-B
(c) i-D, v- D, iii- A, iv-B
(d) i-A, ii- C, iii- D, iv-B

51. Which of the following analogies is correct:


(a)Nitrogen: 1s22s22p3 :: Argon:1s22s22p6
(b)Carbon: maximum compounds :: Xenon: no compounds
(c) XeF2: Linear :: ClF3: Trigonal planar
(d)Helium: meteorological observations:: Argon: metallurgical processes

52. Complete the following analogy:


Same molecular formula but different structures: A:: Non superimposable mirror images: B
(a) A:Isomers B: Enantiomer
(b) A: Enantiomers B: Racemic mixture
(c) A: Sterioisomers B: Retention
(d) A: IsomersB: Sterioisomers

CASE1: Read the passage given below and answer the following questions 53-55

Early crystallographers had trouble solving the structures of inorganic solids using X-ray
diffraction because some of the mathematical tools for analyzing the data had not yet been
developed. Once a trial structure was proposed, it was relatively easy to calculate the
diffraction pattern, but it was difficult to go the other way (from the diffraction pattern to the
structure) if nothing was known a priori about the arrangement of atoms in the unit cell. It
was important to develop some guidelines for guessing the coordination numbers and
bonding geometries of atoms in crystals. The first such rules were proposed by Linus Pauling,
who considered how one might pack together oppositely charged spheres of different radii.
Pauling proposed from geometric considerations that the quality of the "fit" depended on
the radius ratio of the anion and the cation.
If the anion is considered as the packing atom in the crystal, then the smaller cation fills
interstitial sites ("holes"). Cations will find arrangements in which they can contact the
largest number of anions. If the cation can touch all of its nearest neighbour anions then the
fit is good. If the cation is too small for a given site, that coordination number will be
unstable and it will prefer a lower coordination structure. The table below gives the ranges of
cation/anion radius ratios that give the best fit for a given coordination geometry.

Coordination Geometry ρ = rcation/ranion


number
2 linear 0 - 0.155
3 triangular 0.155 - 0.225
4 tetrahedral 0.225 - 0.414
4 square planar 0.414 - 0.732
6 octahedral 0.414 - 0.732
8 cubic 0.732 - 1.0
12 cuboctahedral 1.0

(Source: Ionic Radii and Radius Ratios. (2021, June 8). Retrieved June 29, 2021, from
https://chem.libretexts.org/@go/page/183346)

-
Q53. The radius of Ag+ ion is 126pm and of I ion is 216pm. The coordination number of Ag+
ion is:
(a)2
(b)3
(c)6
(d)8
Q54. A solid AB has square planar structure. If the radius of cation A+ is 120pm, calculate
-
the maximum possible value of anion B
(a)240 pm
(b)270 pm
(c)280 pm
(d)290 pm

Q55.A “good fit” is considered to be one where the cation can touch:
(a) all of its nearest neighbour anions.
(b) most of its nearest neighbour anions.
(c) some of its nearest neighbour anions.
(d) none of its nearest neighbour anions.

___________________________________________________________________________
SAMPLE PAPER QUESTION (2021-22)
TERM – II
CHEMISTRY THEORY (043)

MM:35 Time: 2 Hours

GENERAL INSTRUCTIONS:
Read the following instructions carefully.
1. There are 12 questions in this question paper with internal choice.
2. SECTION A - Q. No. 1 to 3 are very short answer questions carrying 2 marks each.
3. SECTION B - Q. No. 4 to 11 are short answer questions carrying 3 marks each.
4. SECTION C- Q. No. 12 is case based question carrying 5 marks.
5. All questions are compulsory.
6. Use of log tables and calculators is not allowed

SECTION A
1. Arrange the following in the increasing order of their property indicated (any 2):
a. Benzoic acid, Phenol, Picric acid, Salicylic acid (pka values).
b. Acetaldehyde, Acetone, Methyl tert butyl ketone (reactivity towards NH 2OH).
c. ethanol, ethanoic acid, benzoic acid (boiling point) (1x2=2)

2. Solutions of two electrolytes ‘A’ and ‘B’ are diluted. The Λm of ‘B’ increases 1.5 times
while that of A increases 25 times. Which of the two is a strong electrolyte? Justify your
answer. Graphically show the behavior of ‘A’ and ‘B’. (2)

3. Give reasons to support the answer:


a. Presence of Alpha hydrogen in aldehydes and ketones is essential for aldol condensation.
b. 3 –Hydroxy pentan-2-one shows positive Tollen’s test. (1x2=2)

SECTION B
4. Account for the following:
a. Aniline cannot be prepared by the ammonolysis of chlorobenzene under normal
conditions.
b. N-ethylethanamine boils at 329.3K and butanamine boils at 350.8K, although both are
isomeric in nature.
c. Acylation of aniline is carried out in the presence of pyridine. (1x3=3)

OR
4. Convert the following:
a. Phenol to N-phenylethanamide.
b. Chloroethane to methanamine.
c. Propanenitrile to ethanol. (1x3=3)
5. Answer the following questions:
a. [Ni(H2O)6 ] 2+ (aq) is green in colour whereas [Ni(H2O)4 (en)]2+(aq)is blue in colour , give
reason in support of your answer .
b. Write the formula and hybridization of the following compound:
tris(ethane-1,2–diamine) cobalt(III) sulphate (1+2)

OR

5. In a coordination entity, the electronic configuration of the central metal ion is t 2g3 eg1
a. Is the coordination compound a high spin or low spin complex?
b. Draw the crystal field splitting diagram for the above complex. (1+2)

6. Account for the following:


a. Ti(IV) is more stable than the Ti (II) or Ti(III).
b. In case of transition elements, ions of the same charge in a given series show progressive
decrease in radius with increasing atomic number.
c. Zinc is a comparatively a soft metal, iron and chromium are typically hard. (1x3=3)

7. An alkene ‘A’ (Mol. formula C5H10) on ozonolysis gives a mixture of two compounds ‘B’
and ‘C’. Compound ‘B’ gives positive Fehling’s test and also forms iodoform on treatment
with I2 and NaOH. Compound ‘C’ does not give Fehling’s test but forms iodoform. Identify
the compounds A, B and C. Write the reaction for ozonolysis and formation of iodoform
from B and C. (3)

8. Observe the figure given below and answer the questions that follow:

a. Which process is represented in the figure?


b. What is the application of this process?
c. Can the same process occur without applying electric field? Why is the electric field
applied?

9. What happens when reactions:


a. N-ethylethanamine reacts with benzenesulphonyl chloride.
b. Benzylchloride is treated with ammonia followed by the reaction with Chloromethane.
c. Aniline reacts with chloroform in the presence of alcoholic potassium hydroxide. (1x3=3)
OR

9. a. Write the IUPAC name for the following organic compound:


CH3 – N—CH2CH3

b.Complete the following:

Sn/HCl Br2/H2O NaNO2/HCl HBF4


C6H5NO2 A B C D
273-278K
(1x3=3)
10. Represent the cell in which the following reaction takes place.The value of E˚ for the cell is
1.260 V. What is the value of Ecell ?

2Al(s) + 3Cd2+ (0.1M)  3Cd (s) + 2Al3+ (0.01M) (3)

11. a. Why are fluorides of transition metals more stable in their higher oxidation state as
compared to the lower oxidation state?
b. Which one of the following would feel attraction when placed in magnetic field: Co 2+ ,
Ag+ ,Ti4+ , Zn2+
c. It has been observed that first ionization energy of 5 d series of transition elements are
higher than that of 3d and 4d series, explain why? (1x3=3)

OR

11. On the basis of the figure given below, answer the following questions:
(source: NCERT)

a. Why Manganese has lower melting point


than Chromium?
b. Why do transition metals of 3d series have lower melting points as compared to 4d
series?
c. In the third transition series, identify and name the metal with the highest melting point.
(1x3=3)
SECTION C

12. Read the passage given below and answer the questions that follow.
Are there nuclear reactions going on in our bodies?

There are nuclear reactions constantly occurring in our bodies, but there are very few of them
compared to the chemical reactions, and they do not affect our bodies much. All of the physical
processes that take place to keep a human body running are chemical processes. Nuclear
reactions can lead to chemical damage, which the body may notice and try to fix.
The nuclear reaction occurring in our bodies is radioactive decay. This is the change of a less
stable nucleus to a more stable nucleus. Every atom has either a stable nucleus or an unstable
nucleus, depending on how big it is and on the ratio of protons to neutrons. The ratio of neutrons
to protons in a stable nucleus is thus around 1:1 for small nuclei (Z < 20). Nuclei with too many
neutrons, too few neutrons, or that are simply too big are unstable. They eventually transform to
a stable form through radioactive decay. Wherever there are atoms with unstable nuclei
(radioactive atoms), there are nuclear reactions occurring naturally. The interesting thing is that
there are small amounts of radioactive atoms everywhere: in your chair, in the ground, in the
food you eat, and yes, in your body.

The most common natural radioactive isotopes in humans are carbon-14 and potassium-40.
Chemically, these isotopes behave exactly like stable carbon and potassium. For this reason, the
body uses carbon-14 and potassium-40 just like it does normal carbon and potassium; building
them into the different parts of the cells, without knowing that they are radioactive. In time,
carbon-14 atoms decay to stable nitrogen atoms and potassium-40 atoms decay to stable calcium
atoms. Chemicals in the body that relied on having a carbon-14 atom or potassium-40 atom in a
certain spot will suddenly have a nitrogen or calcium atom. Such a change damages the
chemical. Normally, such changes are so rare, that the body can repair the damage or filter away
the damaged chemicals.

The natural occurrence of carbon-14 decay in the body is the core principle behind carbon
dating. As long as a person is alive and still eating, every carbon-14 atom that decays into a
nitrogen atom is replaced on average with a new carbon-14 atom. But once a person dies, he
stops replacing the decaying carbon-14 atoms. Slowly the carbon-14 atoms decay to nitrogen
without being replaced, so that there is less and less carbon-14 in a dead body. The rate at which
carbon-14 decays is constant and follows first order kinetics. It has a half - life of nearly 6000
years, so by measuring the relative amount of carbon-14 in a bone, archeologists can calculate
when the person died. All living organisms consume carbon, so carbon dating can be used to date
any living organism, and any object made from a living organism. Bones, wood, leather, and
even paper can be accurately dated, as long as they first existed within the last 60,000 years. This
is all because of the fact that nuclear reactions naturally occur in living organisms.

(source: The textbook Chemistry: The Practical Science by Paul B. Kelter, Michael D. Mosher
and Andrew Scott states)

a. Why is Carbon -14 radioactive while Carbon -12 not? (Atomic number of Carbon: 6)
b. Researchers have uncovered the youngest known dinosaur bone, dating around 65 million
years ago. How was the age of this fossil estimated?
c. Which are the two most common radioactive decays happening in human body?
d. Suppose an organism has 20 g of Carbon -14 at its time of death. Approximately how much
Carbon -14 remains after 10,320 years? (Given antilog 0.517 = 3.289)
OR
d. Approximately how old is a fossil with 12 g of Carbon -14 if it initially possessed 32 g of
Carbon -14? (Given log 2.667 = 0.4260)
(1+1+1+2)
SAMPLE PAPER (2022-23)
CHEMISTRY THEORY
(043)
MM:70 Time: 3 hours
General Instructions:
Read the following instructions carefully.
a) There are 35 questions in this question paper with internal choice.
b) SECTION A consists of 18 multiple-choice questions carrying 1 mark each.
c) SECTION B consists of 7 very short answer questions carrying 2 marks each.
d) SECTION C consists of 5 short answer questions carrying 3 marks each.
e) SECTION D consists of 2 case- based questions carrying 4 marks each.
f) SECTION E consists of 3 long answer questions carrying 5 marks each.
g) All questions are compulsory.
h) Use of log tables and calculators is not allowed

SECTION A

The following questions are multiple-choice questions with one correct answer. Each
question carries 1 mark. There is no internal choice in this section.

1. The major product of acid catalysed dehydration of 1-methylcyclohexanol is:


a. 1-methylcyclohexane
b. 1-methylcyclohexene
c. 1-cyclohexylmethanol
d. 1-methylenecyclohexane

2. Which one of the following compounds is more reactive towards SN1 reaction?
a. CH2=CHCH2Br
b. C6H5CH2Br
c. C6H5CH (C6H5)Br
d. C6H5CH(CH3) Br

3. KMnO4 is coloured due to:


a. d-d transitions
b. charge transfer from ligand to metal
c. unpaired electrons in d orbital of Mn
d. charge transfer from metal to ligand

1
4. Which radioactive isotope would have the longer half- life 15O or 19O? (Given rate
constants for 15O and 19O are 5.63x 10-3 s-1 and k = 2.38 x 10-2s-1 respectively.)

a. 15O
b. 19O
c. Both will have the same half-life
d. None of the above, information given is insufficient

5. The molar conductivity of CH3COOH at infinite dilution is 390 Scm2/mol. Using the
graph and given information, the molar conductivity of CH3COOK will be:

a. 100 Scm2/mol
b. 115 Scm2/mol
c. 150 Scm2/mol
d. 125 Scm2/mol

*FOR VISUALLY CHALLENGED LEARNERS

*5. What is the molar conductance at infinite dilution for sodium chloride if the molar
conductance at infinite dilution of Na+ and Cl- ions are 51.12 × 10-4 Scm2/mol and
73.54× 10-4 Scm2/mol respectively?

a. 124.66 Scm2/mol
b. 22.42 Scm2/mol
c. 198.20Scm2/mol
d. 175.78 Scm2/mol

2
6. For the reaction, A +2B  AB2, the order w.r.t. reactant A is 2 and w.r.t. reactant B.
What will be change in rate of reaction if the concentration of A is doubled and B is
halved?

a. increases four times


b. decreases four times
c. increases two times
d. no change

7. Arrange the following in the increasing order of their boiling points:


A : Butanamine, B: N,N-Dimethylethanamine, C: N- Etthylethanaminamine

a. C<B<A
b. A<B<C
c. A<C<B
d. B<C<A

8. The CFSE of [CoCl6]3- is 18000 cm-1 the CFSE for [CoCl4]- will be:
a. 18000 cm-1
b. 8000cm-1
c. 2000 cm-1
d. 16000 cm-1

9. What would be the major product of the following reaction?


C6H5 -CH2-OC6H5 + HBr  A + B

a. A= C6H5CH2OH , B= C6H6
b. A=C6H5CH2OH ,B= C6H5Br
c. A=C6H5CH3 ,B= C6H5Br
d. A=C6H5CH2Br , B= C6H5OH

10. Which of the following statements is not correct for amines?


a. Most alkyl amines are more basic than ammonia solution.
b. pKb value of ethylamine is lower than benzylamine.
c. CH3NH2 on reaction with nitrous acid releases NO2 gas.
d. Hinsberg’s reagent reacts with secondary amines to form sulphonamides.

11. Which of the following tests/ reactions is given by aldehydes as well as ketones?
a. Fehling’s test
b. Tollen’s test
c. 2,4 DNP test
d. Cannizzaro reaction

3
12. Arrhenius equation can be represented graphically as follows:

The (i) intercept and (ii) slope of the graph are:


a. (i) ln A (ii) Ea/R
b. (i) A (ii) Ea
c. (i)ln A (ii) - Ea/R
d. (i) A (ii) -Ea

*FOR VISUALLY CHALLENGED LEARNERS

*12. The unit of rate constant for the reaction


2A + 2B → A2B2
which has rate = k [A]2[B] is:
a. mol L-1s-1
b. s-1
c. mol L-1
d. mol-2 L2 s-1

13. The number of ions formed on dissolving one molecule of FeSO4.(NH4)2SO4.6H2O


in water is:

a. 3
b. 4
c. 5
d. 6

14. The oxidation of toluene to benzaldehyde by chromyl chloride is called


a. Etard reaction
b. Riemer-Tiemann reaction
c. Stephen’s reaction
d. Cannizzaro’s reaction

15. Given below are two statements labelled as Assertion (A) and Reason (R)

4
Assertion (A): An ether is more volatile than an alcohol of comparable molecular
mass.
Reason (R): Ethers are polar in nature.
Select the most appropriate answer from the options given below:
a. Both A and R are true and R is the correct explanation of A
b. Both A and R are true but R is not the correct explanation of A.
c. A is true but R is false.
d. A is false but R is true.

16. Given below are two statements labelled as Assertion (A) and Reason (R)

Assertion (A): Proteins are found to have two different types of secondary
structures viz alpha-helix and beta-pleated sheet structure.
Reason (R): The secondary structure of proteins is stabilized by hydrogen
bonding.
Select the most appropriate answer from the options given below:
a. Both A and R are true and R is the correct explanation of A
b. Both A and R are true but R is not the correct explanation of A.
c. A is true but R is false.
d. A is false but R is true.

17. Given below are two statements labelled as Assertion (A) and Reason (R)

Assertion : Magnetic moment values of actinides are lesser than the


theoretically predicted values.
Reason : Actinide elements are strongly paramagnetic.
Select the most appropriate answer from the options given below:
a. Both A and R are true and R is the correct explanation of A
b. Both A and R are true but R is not the correct explanation of A.
c. A is true but R is false.
d. A is false but R is true.

18. Given below are two statements labelled as Assertion (A) and Reason (R)

Assertion (A): Tertiary amines are more basic than corresponding secondary
and primary amines in gaseous state.
Reason (R): Tertiary amines have three alkyl groups which cause +I effect.
Select the most appropriate answer from the options given below:
a. Both A and R are true and R is the correct explanation of A
b. Both A and R are true but R is not the correct explanation of A.
c. A is true but R is false.
d. A is false but R is true.

SECTION B

5
This section contains 7 questions with internal choice in two questions. The following
questions are very short answer type and carry 2 marks each.

19. A first-order reaction takes 69.3 min for 50% completion. What is the time needed for
80% of the reaction to get completed?
(Given: log 5 =0.6990, log 8 = 0.9030, log 2 = 0.3010)

20. Account for the following:


a. There are 5 OH groups in glucose
b. Glucose is a reducing sugar

OR

What happens when D – glucose is treated with the following reagents


a. Bromine water
b. HNO3

21. Give reason for the following:


a. During the electrophilic substitution reaction of haloarenes, para
substituted derivative is the major product.
b. The product formed during SN1 reaction is a racemic mixture.

OR

a. Name the suitable alcohol and reagent, from which 2-Chloro-2-methyl


propane can be prepared.
b. Out of the Chloromethane and Fluoromethane , which one is has higher
dipole moment and why?

22. The formula Co(NH3)5CO3Cl could represent a carbonate or a chloride. Write the
structures and names of possible isomers.

23. Corrosion is an electrochemical phenomenon. The oxygen in moist air reacts as


follows:

O2(g) + 2H2O(l) + 4e– → 4OH– (aq).

Write down the possible reactions for corrosion of zinc occurring at anode, cathode, and
overall reaction to form a white layer of zinc hydroxide.

24. Explain how and why will the rate of reaction for a given reaction be affected when
a. a catalyst is added
b. the temperature at which the reaction was taking place is decreased

25. Write the reaction and IUPAC name of the product formed when 2-Methylpropanal
(isobutyraldehyde) is treated with ethyl magnesium bromide followed by hydrolysis.

6
SECTION C

This section contains 5 questions with internal choice in two questions. The following
questions are short answer type and carry 3 marks each.

26. Write the equations for the following reaction:


a. Salicylic acid is treated with acetic anhydride in the presence of conc.
H2SO4
b. Tert butyl chloride is treated with sodium ethoxide.
c. Phenol is treated with chloroform in the presence of NaOH

27. Using Valence bond theory, explain the following in relation to the paramagnetic
complex [Mn(CN)6]3-
a. type of hybridization
b. magnetic moment value
c. type of complex – inner, outer orbital complex

28. Answer the following questions:


a. State Henry’s law and explain why are the tanks used by scuba divers
filled with air diluted with helium (11.7% helium, 56.2% nitrogen and
32.1% oxygen)?
b. Assume that argon exerts a partial pressure of 6 bar. Calculate the
solubility of argon gas in water. (Given Henry’s law constant for argon
dissolved in water, KH = 40kbar)

29. Give reasons for any 3 of the following observations:


a. Aniline is acetylated before nitration reaction.
b. pKb of aniline is lower than the m-nitroaniline.
c. Primary amine on treatment with benzenesulphonyl chloride forms a
product which is soluble in NaOH however secondary amine gives product
which is insoluble in NaOH.
d. Aniline does not react with methyl chloride in the presence of anhydrous
AlCl3 catalyst.

30. a. Identify the major product formed when 2-cyclohexylchloroethane


undergoes a dehydrohalogenation reaction. Name the reagent which is
used to carry out the reaction.
b. Why are haloalkanes more reactive towards nucleophilic substitution
reactions than haloarenes and vinylic halides?

OR

a. Name the possible alkenes which will yield 1-chloro-1-methylcyclohexane


on their reaction with HCl. Write the reactions involved.
b. Allyl chloride is hydrolysed more readily than n-propyl chloride. Why?

7
SECTION D

The following questions are case-based questions. Each question has an internal
choice and carries 4 (1+1+2) marks each. Read the passage carefully and answer the
questions that follow.

31. Strengthening the Foundation: Chargaff Formulates His "Rules"


Many people believe that James Watson and Francis Crick discovered DNA in the
1950s. In reality, this is not the case. Rather, DNA was first identified in the late
1860s by Swiss chemist Friedrich Miescher. Then, in the decades following
Miescher's discovery, other scientists--notably, Phoebus Levene and Erwin
Chargaff--carried out a series of research efforts that revealed additional details
about the DNA molecule, including its primary chemical components and the ways
in which they joined with one another. Without the scientific foundation provided
by these pioneers, Watson and Crick may never have reached their
groundbreaking conclusion of 1953: that the DNA molecule exists in the form of a
three-dimensional double helix.
Chargaff, an Austrian biochemist, as his first step in this DNA research, set out to
see whether there were any differences in DNA among different species. After
developing a new paper chromatography method for separating and identifying
small amounts of organic material, Chargaff reached two major conclusions:
(i) the nucleotide composition of DNA varies among species.
(ii) Almost all DNA, no matter what organism or tissue type it comes from maintains
certain properties, even as its composition varies. In particular, the amount of
adenine (A) is similar to the amount of thymine (T), and the amount of guanine (G)
approximates the amount of cytosine (C). In other words, the total amount of
purines (A + G) and the total amount of pyrimidines (C + T) are usually nearly
equal. This conclusion is now known as "Chargaff's rule."
Chargaff’s rule is not obeyed in some viruses. These either have single- stranded
DNA or RNA as their genetic material.

Answer the following questions:

a. A segment of DNA has 100 adenine and 150 cytosine bases. What is the
total number of nucleotides present in this segment of DNA?
b. A sample of hair and blood was found at two sites. Scientists claim that the
samples belong to same species. How did the scientists arrive at this
conclusion?
c. The sample of a virus was tested and it was found to contain 20% adenine,
20% thymine, 20 % guanine and the rest cytosine. Is the genetic material
of this virus (a) DNA- double helix (b) DNA-single helix (c) RNA? What do
you infer from this data?

8
OR

How can Chargaff’s rule be used to infer that the genetic material of an
organism is double- helix or single- helix?

32. Henna is investigating the melting point of different salt solutions.


She makes a salt solution using 10 mL of water with a known mass of NaCl salt.
She puts the salt solution into a freezer and leaves it to freeze.
She takes the frozen salt solution out of the freezer and measures the
temperature when the frozen salt solution melts.
She repeats each experiment.

S.No Mass of the salt Melting point in 0C


used in g
Readings Set 1 Reading Set 2
1 0.3 -1.9 -1.9
2 0.4 -2.5 -2.6
3 0.5 -3.0 -5.5
4 0.6 -3.8 -3.8
5 0.8 -5.1 -5.0
6 1.0 -6.4 -6.3

Assuming the melting point of pure water as 0oC, answer the following
questions:
a. One temperature in the second set of results does not fit the pattern.
Which temperature is that? Justify your answer.
b. Why did Henna collect two sets of results?
c. In place of NaCl, if Henna had used glucose, what would have been the
melting point of the solution with 0.6 g glucose in it?

OR
What is the predicted melting point if 1.2 g of salt is added to 10 mL of
water? Justify your answer.

SECTION E

The following questions are long answer type and carry 5 marks each. Two questions
have an internal choice.

33. a. Why does the cell voltage of a mercury cell remain constant during its

9
lifetime?
b. Write the reaction occurring at anode and cathode and the products of
electrolysis of aq KCl.
c. What is the pH of HCl solution when the hydrogen gas electrode shows
a potential of -0.59 V at standard temperature and pressure?

OR

a. Molar conductivity of substance “A” is 5.9×103 S/m and “B” is 1 x 10-16


S/m. Which of the two is most likely to be copper metal and why?
b. What is the quantity of electricity in Coulombs required to produce 4.8 g of
Mg from molten MgCl2? How much Ca will be produced if the same
amount of electricity was passed through molten CaCl2? (Atomic mass of
Mg = 24 u, atomic mass of Ca = 40 u).
c. What is the standard free energy change for the following reaction at room
temperature? Is the reaction spontaneous?

Sn(s) + 2Cu2+ (aq) à Sn2+ (aq) + 2Cu+ (s)

34. A hydrocarbon (A) with molecular formula C5H10 on ozonolysis gives two products
(B) and ( C). Both (B) and (C) give a yellow precipitate when heated with iodine in
presence of NaOH while only (B) give a silver mirror on reaction with Tollen’s
reagent.
a. Identify (A), (B) and (C).
b. Write the reaction of B with Tollen’s reagent
c. Write the equation for iodoform test for C
d. Write down the equation for aldol condensation reaction of B and C.

OR

An organic compound (A) with molecular formula C2Cl3O2H is obtained when (B)
reacts with Red P and Cl2. The organic compound (B) can be obtained on the
reaction of methyl magnesium chloride with dry ice followed by acid hydrolysis.
a. Identify A and B
b. Write down the reaction for the formation of A from B. What is this reaction
called?
c. Give any one method by which organic compound B can be prepared from
its corresponding acid chloride.
d. Which will be the more acidic compound (A) or (B)? Why?
e. Write down the reaction to prepare methane from the compound (B).
35. Answer the following:
a. Why are all copper halides known except that copper iodide?
b. Why is the Eo(V /V ) value for vanadium comparatively low?
3+ 2+

c. Why HCl should not be used for potassium permanganate titrations?


10
d. Explain the observation, at the end of each period, there is a slight
increase in the atomic radius of d block elements.
e. What is the effect of pH on dichromate ion solution?

11
SAMPLE PAPER (2023 -24)
CHEMISTRY THEORY (043)
M ax. M arks:70 Time: 3 hours
General Instructions:
Read the following instructions carefully.

(a) There are 33 questions in this question paper with internal choice.
(b) SECTION A consists of 16 multiple -choice questions carrying 1 mark each.
(c) SECTION B consists of 5 short answer questions carrying 2 marks each.
(d) SECTION C consists of 7 short answer questions carrying 3 marks each.
(e) SECTION D consists of 2 case - based questions carrying 4 marks each.
(f) SECTION E consists of 3 long answer questions carrying 5 marks each.
(g) All questions are compulsory.
(h) Use of log tables and calculators is not allowed.

SECTION A
The following questions are multiple -choice questions with one correct answer. Each question carries 1
mark. There is no internal choice in this section.

1. Which of the following solutions will have the highest conductivity at 298 K?
(a) 0.01 M HCl solution (b) 0.1 M HCl solution
(c) 0.01 M CH 3COOH solution (d) 0.1 M CH 3COOH solution

2.

Identify A and B:
(a) A = 1-phenylethanal , B = acetophenone (b) A = Benzophenone B = formaldehyde
(c) A= Benzaldehyde , B = Acetophenone (d) A = Benzophenone , B = Acetophenone
3. The vitamins which can be stored in our body are:
(a) Vitamin A, B, D and E (d) Vitamin A, C, D and K
(c) Vitamin A, B, C and D (d) Vitamin A, D, E and K
4. What is IUPAC name of the ketone A, which undergoes iodo form reaction to give
CH 3 CH= C(CH 3)COONa and yellow precipitate of CHI 3 ?
(a) 3-Methylpent-3-en-2one (b) 3-Methylbut-2-en- one
(c) 2, 3-Dimethylethanone (d) 3-Methylpent-4-one
5. Which of the following is not correct?
(a) In haloarenes, the electron pairs on halogen atom are in conjugation with π-electrons of the ring.
(b) The carbon-magnesium bond is covalent and non-polar in nature .
(c) During SN1 reaction, the carbocation formed in the slow step being sp2 hybridised is planar.
(d) Out of CH 2= CH-Cl and C6H 5CH 2Cl, C6H 5CH 2Cl is more reactive towards SN1 reaction
6. Match the properties with the elements of 3d series:
(i) lowest enthalpy of atomisation (p) Sc
(ii) shows maximum number of oxidation states (q) Mn
(iii) transition metal that does not form coloured compounds (r) Zn
(s) Ti
(a) (i) (r), (ii) (q), (iii) (p) (b) (i) (r), (ii) (s), (iii) (p)
(c) (i) (p), (ii) (q), (iii) (r) (d) (i) (s), (ii) (r), (iii) (p)
7. Which of the following statement is true?
(a) molecularity of reaction can be zero or a fraction.
(b) molecularity has no meaning for complex reactions.
(c) molecularity of a reaction is an experimental quantity
(d) reactions with the molecularity three are very rare but are fast.
8. In which of the following solvents, the C4H 8NH 3+X– is soluble;
(a) ether (b) acetone (c) water (d) bromine water
9. Which of the following observation is shown by 2 -phenyl ethanol with Lucas Reagent?
(a) Turbidity will be observed within five minutes
(b) No turbidity will be observed
(c) Turbidity will be observed immediately
(d) Turbidity will be observed at room temperature but will disappear after five minutes.
10. If the initial concentration of substance A is 1.5 M and after 120 seconds the concentration of
substance A is 0.75 M, the rate constant for the reaction if it follows zero - order kinetics is:
(a) 0.00625 molL-1s-1 (b) 0.00625 s-1 (c) 0.00578 molL-1s-1 (d) 0.00578 s-1
11. Anisole undergoes bromination with bromine in ethanoic acid even in the absence of iron (III)
bromide catalyst
(a) Due to the activation of benzene ring by the methoxy group.
(b) Due to the de-activation of benzene ring by the methoxy group.
(c) Due to the increase in electron density at ortho and para positions
(d) Due to the formation of stable carbocation .
12 . The trend of which property is represented by the following graph?

(a) ionization enthalpy (b) atomic radii


(c) enthalpy of atomization (d) melting point
For Visually Challenged Learners

12. Which of the following is not considered a transition element?


(a) Scandium (b) Silver (c) Vanadium (d) Zinc
13. Given below are two statements labelled as Assertion (A) and Reason (R)
Assertion (A): Alcohols react both as nucleophiles and electrophiles.
Reason (R): The bond between C–O is broken when alcohols react as nucleophiles.
Select the most appropriate answer from the options given below:
(a) Both A and R are true and R is the correct explanation of A
(b) Both A and R are true but R is not the correct explanation of A.
(c) A is true but R is false.
(d) A is false but R is true.
14. Given below are two statements labelled as Assertion (A) and Reason (R)
Assertion (A): Strong oxidising agents oxidise toluene and its derivatives to benzoic acids.
Reason (R): It is possible to stop the oxidation of toluene at the aldehyde stage with suitable
reagents.
Select the most appropriate answer from the options given below:
(a) Both A and R are true and R is the correct explanation of A
(b) Both A and R are true but R is not the correct explanation of A.
(c) A is true but R is false.
(d) A is false but R is true.
15. Given below are two statements labelled as Assertion (A) and Reason (R)
Assertion : Enzymes are very specific for a particular reaction and for a particular substrate.
Reason : Enzymes are biocatalysts.
Select the most appropriate answer from the options given below:
(a) Both A and R are true and R is the correct explanation of A
(b) Both A and R are true but R is not the correct explanation of A.
(c) A is true but R is false.
(d) A is false but R is true.
16. Given below are two statements labelled as Assertion (A) and Reason (R)
Assertion (A): During electrolysis of aqueous copper sulphate solution using copper electrodes
hydrogen gas is released at the cathode.
Reason (R): The electrode potential of Cu2+ /Cu is greater than that of H +/H 2
Select the most appropriate answer from the options given below:
(a) Both A and R are true and R is the correct explanation of A
(b) Both A and R are true but R is not the correct explanation of A.
(c) A is true but R is false.
(d) A is false but R is true.

3
SECTION B
This section contains 5 questions with internal choice in one question. The following questions are
very short answer type and carry 2 marks each.

17. a. Radioactive decay follows first - order kinetics. The initial amount of two radioactive
elements X and Y is 1 gm each. What will be the ratio of X and Y after two days if their half-
lives are 12 hours and 16 hours respectively?
b. The hypothetical reaction P + Q R is half order w.r.t ‘P’ and zero order w.r.t ‘Q’. What
is the unit of rate constant for this reaction?

18. A 5% solution of Na2SO4.10H 2O (MW = 3 22) is isotonic with 2% solution of non- electrolytic, non
volatile substance X. Find out the molecular weight of X.

19. (a) Arrange the isomeric dichlorobenzene in the increasing order of their boiling point and melting
points.

(b) Explain why the electrophilic substitution reactions in haloarenes occur slowly and require
more drastic conditions as compared to those in benzene.
20. (a) Out of p-tolualdehyde and p-nitrobenzaldehyde ,which one is more reactive towards
nucleophilic addition reactions, why?

(b) Write the structure of the product formed when acetone reacts with 2,4 DNP reagent .
OR

Convert the following:


(a) Benzene to m-nitrobenzaldehyde

(b) Bromobenzene to benzoic acid


21. (a) DNA fingerprinting is used to determine paternity of an individual. Which property of DNA
helps in the procedure?
(b) What structural change will occur when a native protein is subjected to change in pH?

SECTION C
This section contains 7 questions with internal choice in one question. The following questions are
short answer type and carry 3 marks each.

22. (a) Write the formula for the following coordination compound

Bis(ethane-1,2-diamine) dihydroxidochromium(III) chloride

(b) Does ionization isomer for the following compound exist? Justify your answer.

Hg[Co(SCN) 4]

(c) Is the central metal atom in coordination complexes a Lewis acid or a Lewis base? Explain.

23. (a) Can we construct an electrochemical cell with two half-cells composed of ZnSO 4 solution and
zinc electrodes? Explain your answer.

(b) Calculate the λ0m for Cl- ion from the data given below:

Λ0m MgCl2 = 258.6 Scm2mol–1 and λ0m Mg2+ = 106 Scm2mol–1

(c) The cell constant of a conductivity cell is 0.146 cm -1. What is the conductivity of 0.01 M solution
of an electrolyte at 298 K, if the resistance of the cell is 1000 ohm?
4
24. Write the name of the reaction, structure and IUPAC name of the product formed when:

(a) phenol reacts with CHCl3 in the presence of NaOH followed by hydrolysis.
(b) CH 3CH2CH(CH 3)CH(CH 3)ONa reacts with C2H 5Br
25. You are given four organic compounds “A”, “B” , “C” and “D”. The compounds “A”, “B” and “C”
form an orange- red precipitate with 2,4 DNP reagent. Compounds “A” and “B” reduce Tollen’s
reagent while compounds “C” and “D” do not. Both “B” and “C” give a yellow precipitate when
heated with iodine in the presence of NaOH. Compound “D” gives brisk effervescence with sodium
bicarbonate solution. Identify “A”, “B”, “C” and “D” given the number of carbon atoms in three of
these carbon compounds is three while one has two carbon atoms. Give an explanation for y our
answer.
26. When sucrose is hydrolysed the optical rotation values are measured using a polarimeter and are
given in the following table:

S.No. Time (hours) Specific Rotation

1 0 + 66.5o
2 ∞ -39.9o

(a) Account for the two specific rotation values.


(b) What is the specific name given to sucrose based on the above observation .
(c) One of the products formed during the hydrolysis of sucrose is a glucose, that reacts with
hydroxylamine to give compound A. Identify compound A.

27. An organic compound A with the molecular formula (+)C 4H 9Br undergoes hydrolysis to form (+ )
C4H 9OH. Give the structure of A and write the mechanism of the reaction.
28. The rate constants of a reaction at 200K and 500K are 0.02s–1 and 0.20s–1 respectively. Calculate the
value of Ea (Given 2.303R = 19.15 JK -1mol-1)

SECTION D
The following questions are case -based questions. Each question has an internal choice and carries
4 (1+1+2) marks each. Read the passage carefully and answer the questions that follow.
29. Crystal field splitting by various ligands
Metal complexes show different colours due to d-d transitions. The complex absorbs light of specific
wavelength to promote the electron from t2g to eg level. The colour of the complex is due to the
transmitted light, which is complementary of the colour absorbed.
The wave number of light absorbed by different complexes of Cr ion are given below:

Complex Wavenumber of light Energy of light absorbed


absorbed (cm-1) (kJ/mol)
[CrA6]3- 13640 163
[CrB6]3+ 17830 213
[CrC6]3+ 21680 259
[CrD6]3- 26280 314

5
Answer the following questions:

(a) Out of the ligands “A”, “B”, “C” and “D”, which ligand causes maximum crystal field splitting?
Why?
OR

Which of the two, “A” or “D” will be a weak field ligand? Why?

(b) Which of the complexes will be violet in colour? [CrA 6]3- or [CrB6]3+ and why? (Given: If 560 -
570 nm of light is absorbed, the colour of the complex observed is violet.)

(c) If the ligands attached to Cr3+ ion in the complexes given in the table above are water, cyanide
ion, chloride ion, and ammonia (not in this order)

Identify the ligand, write the formula and IUPAC name of the following:
(i) [CrA 6]3- (ii) [CrC6]3+
30. The lead-acid battery represents the oldest rechargeable battery technology. Lead acid batteries can
be found in a wide variety of applications including small-scale power storage such as UPS systems,
ignition power sources for automobiles, along with large, grid-scale power systems. The spongy
lead act as the anode and lead dioxide as the cathode. Aqueous sulphuric acid is used as an
electrolyte. The half-reactions during discharging of lead storage cells are:

Anode: Pb(s)  SO 4 (aq)  PbSO 4 (s)  2e 


2 

Cathode: PbO (s)


2 4H (aq)  SO24 (aq)  2ePbSO (s)
4  2H 2O
There is no safe way of disposal and these batteries end - up in landfills. Lead and sulphuric
acid are extremely hazardous and pollute soil, water as well as air. Irrespective of the
environmental challenges it poses, lead-acid batteries have remained an important source of
energy.
Designing green and sustainable battery systems as alternatives to conventional means
remains relevant. Fuel cells are seen as the future source of energy. Hydrogen is considered a
green fuel. Problem with fuel cells at present is the storage of hydrogen. Currently, ammonia
and methanol are being used as a source of hydrogen for fuel cell. These are obtained
industrially, so add to the environmental issues.
If the problem of storage of hydrogen is overcome, is it still a “green fuel?” Despite being the
most abundant element in the Universe, hydrogen does not exist on its own so needs to be
extracted from the water using electrolysis or separated from carbon fossil fuels. Both of
these processes require a significant amount of energy which is currently more than that
gained from the hydrogen itself. In addition, this extraction typically requires the use of fossil
fuels. More research is being conducted in this field to solve these problems. Despite the
problem of no good means to extract Hydrogen, it is a uniquely abundant and renewable
source of energy, perfect for our future zero-carbon needs.
Answer the following questions:
(a) How many coulombs have been transferred from anode to cathode in order to consume one
mole of sulphuric acid during the discharging of lead storage cell?
(b) How much work can be extracted by using lead storage cell if each cell delivers about
2.0 V of voltage? (1 F = 96500 C)
(c) Do you agree with the statement – “Hydrogen is a green fuel.” Give your comments for and
against this statement and justify your views.
OR
6
Imagine you are a member of an agency funding scientific research. Which of the following
projects will you fund and why?
(i) safe recycling of lead batteries
(ii) extraction of hydrogen

SECTION E
The following questions are long answer type and carry 5 marks each. All questions have an
internal choice.

31. Attempt any five of the following:

(a) Which of the following ions will have a magnetic moment value of 1.73 BM.
Sc3+, Ti3+, Ti2+, Cu2+, Zn2+
(b) In order to protect iron from corrosion, which one will you prefer as a sacrificial electrode, Ni or
Zn? Why? (Given standard electrode potentials of Ni, Fe and Zn are -0.25 V, -0.44 V and -0.76 V
respectively.)
(c) The second ionization enthalpies of chromium and manganese are 1592 and 1509 kJ/mol
respectively. Explain the lower value of Mn.

(d) Give two similarities in the properties of Sc and Zn.


(e) What is actinoid contraction? What causes actinoid contraction?
(f) What is the oxidation state of chromium in chromate ion and dichromate ion?

(g) Write the ionic equation for reaction of KI with acidified KMnO 4.
32. (a) What is the effect of temperature on the solubility of glucose in water?
(b) Ibrahim collected a 10mL each of fresh water and ocean water. He observed that one sample
labeled “P” froze at 0 oC while the other “Q” at -1.3oC. Ibrahim forgot which of the two, “P” or
“Q” was ocean water. Help him identify which container contains ocean water, giving
rationalization for your answer.

(c) Calculate Van't Hoff factor for an aqueous solution of K 3 [Fe(CN)6] if the degree of dissociation
(α) is 0.852. What will be boiling point of this solution if its concentration is 1 molal? (Kb=0.52 K
kg/mol)
OR

(a) What type of deviation from Roult’s Law is expected when phenol and aniline are mixed with
each other? What change in the net volume of the mixture is expected? Graphically represent
the deviation.
(b) The vapour pressure of pure water at a certain temperature is 23.80 mm Hg. If 1 mole of a non-
volatile non- electrolytic solute is dissolved in 100g water, Calculate the resultant vapour
pressure of the solution.

33. An organic compound with molecular formula C7H7NO2 exists in three isomeric forms, the
isomer ‘A’ has the highest melting point of the three. ‘A’ on reduction gives compound ‘B’ with
molecular formula C7H9N. ‘B’ on treatment with NaNO2/HCl at 0-5 0C to form compound ‘C’.
On treating C with H3PO2 ,it gets converted to D with formula C7H8 , which on further reaction
with CrO2Cl2 followed by hydrolysis forms ‘E’ C7H6O . Write the structure of compounds A to
E . Write the chemical equations involved.
OR
7
(a) Account for the following:
(i) N-ethylbenzenesulphonyl amide is soluble in alkali .
(ii) Reduction of nitrobenzene using Fe and HCl is preferred over Sn and HCl.
(b) Arrange the following in:
(i) decreasing order of pKb values
C6H5NH2, C6H5NHCH3, C6H5CH2NH2, CH3NH2, NH3
(ii) increasing order of solubility in water
C2H5Cl, C2H5NH2, C2H5OH
(iii) decreasing boiling point
CH3COOH, C2H5OH, CH3NH2, CH3OCH3

------------------------------------------------------------------------------------

You might also like